Finance Accounts Interview Questions

Finance Accounts Interview Questions
1.How would you feel your skills garnered in college will help you in the professional world?
In the university, I have been exposed to a number of different accounting & finance systems e.g. EasyFinance, SmartNumber, UberAccount, NetSuite, SAP and a few others. Not only were we, the students, exposed to the theory – we also had some deep practical experience in them.
While the name and proprietary of each system differs, essentially they all require a set of good knowledge and expertise in accounting and finance subjects. So I would not have much issue in adapting to distinguished system because I know my basic accounting exposure will prepare me for the working and professional world.
2.What are your greatest strengths you will bring to the company?

I am fast and meticulous – an important combination of attributes for this line of work. Coupled with my competency and speedy adaptation to new environment, I can assure you someone who would make a difference in your organization. I have proven this in the past; I know I will do it again.
Additionally, I have outstanding interpersonal skills, which would benefit this organization in this service oriented business. I develop and maintain rapport with team mates and customers seamlessly, developing respect internally and externally for the company.
3.In your opinion, what would be the main challenges in your role, if you take up this job?
I have been made understand before that the SAP system is in the process of implementation now. Since not everyone in the team is well verse in this system, training and development would be a major focus. I foresee my main challenge, as a certified SAP practitioner, to be leveraging the knowledge and make sure adequate support to the team members.
However, while this can be a challenge, I believe in a couple of months, we will start seeing the benefits of the implementation. The robustness of the SAP system is a major plus, which I believe can help the company streamlining and consolidating the business data with ease and convenience.
4.In this line of work, what makes a good superior?
In my opinion, in any line of work, we need an inspiring leader or superior. He or she may have different style of leading and giving instructions, but the important thing is he knows how to drive the subordinates to loving what they are doing, accomplishing all the tasks and deliver the job beyond the expectations. Winning teams in common have inspiring leaders.
5. What makes you choose finance/accounting?
I love numbers. Not only that, I am good in them. My academic results are proofs of my capability in this line. I know I can grow really big in this career.
6.What interests you about this position?
This is one step ahead of my current position. I have been working with the current company for five years now. With the amount of experience and exposure, I believe the time is right for me to make a move. In other words, I am prepared to take up a more leadership role.
7.It seems the finance team in your current organization is bigger than us. Why you choose this company?
First, I believe growth does not necessarily involve the size of a team or department. It’s about challenges and how you cope with them. Also, I have been following up closely with the development of this company and let me say that I never underestimate this company’s potential. What I have is admiration.
Even though it’s only been in this country for only about 5 years, its long time existence for over half a century deserves some respect. I know that this company can only grow in the right direction.
8.What are important criteria for your next job?
I consider myself someone who is not contented with only the routine work. I would like my potential to be challenged, fulfilled and realized. What important is, I get thrown with new challenges, and given a chance to prove what I can do best.
9.In finance/accounting, is professional qualification such as ACCA considered better than an MBA?
That depends on how we foresee ourselves to grow. Professional qualifications such as ACCA, CIMA is important to stamp ourselves as authority in the subject matter and will be even more important when we want to start our own accounting practice.
An MBA is on the other hand, prepares us for leadership position in major organizations and with the right recipe, one day, we can hold key position such as CEO or MD of public listed companies and may possibly make a lateral move from one industry to another.
10.How would you handle a new implemented finance/accounting system, of which you have no knowledge before?
There has to be a start to something new. To be honest, I am all excited to learn and acquire new skills and knowledge of any new accounting and finance system. The important thing is, I am given adequate time to adapt and learn.
I believe that these days, new system implementation comes with at least some basic training. This would be helpful, but with the right amount of time and space, I will be able to be competent and master this new system.

SBI PO General awarness paper

SBI PO General awarness paper
1.CAS is associated with which of the following ?
(A) Legal System Reforms
(B) Piped gas line
(C) Cable T.V.
(D) Mobile phone regulation
(E) None of these
2.GNP stands for_________
(A) Gross National Product
(B) Group Net Product
(C) Grand Nuclear Process
(D) Group Networking Process
(E) None of these
3.‘Heavy Water’ is used in which of the following types of indus-tries ?
(A) Sugar
(B) Nuclear Power
(C) Textile
(D) Coal
(E) None of these
4.‘Handshaking’ in Networking parlance means
(A) connecting computers to a hub
(B) distributed Networks
(C) having same operating system on different computers
(D) sending e-mail
5.What is the population of India ?
(A) 98 crores
(B) More than 2 billion
(C) More than 1 billion
(D) Less than 96 crores
(E) 96 crores
6.Baichung Bhutia whose name was in news is a well known__________
(A) Music Director of Indian films
(B) English author of Indian origin
(C) Journalist
(D) Politician
(E) Sports Personality
7.Who amongst the following is the author of the book ‘Indomi-table Spirit’?
(A) Dr. A.P.J. Abdul Kalam
(B) Dr. Manmohan Singh
(C) Justice K. G. Balakrishna
(D) Mr. Natwar Singh
(E) None of these
8.‘Richter Scale’ is used to measure which of the following ?
(A) Intensity of Tsunami Waves
(B) Intensity of Earthquake
(C) Density of salt in Sea water
(D) Flow of electric current
(E) None of these
9.Thermostat is an instrument used to_________
(A) measure flow of current
(B) measure intensity of voltage
(C) regulate temperature
(D) regulate velocity of sound
(E) None of these
10.‘Acoustics’ is the science of the study of_________
(A) Light (B) Sound
(C) Electricity (D) Magnetism
(E) None of these
11.Tata Steel recently acquired the Corus, a steel giant situated in_____
(A) South Africa
(B) Ukraine
(C) Australia
(D) Italy
(E) Britain
12.‘Yen’ is the currency of_________
(A) South Korea
(B) China
(C) Indonesia
(D) Malaysia
(E) None of these
13.Who amongst the following is the author of the book ‘Indomi-table Spirit’?
(A) Dr. A.P.J. Abdul Kalam
(B) Dr. Manmohan Singh
(C) Justice K. G. Balakrishna
(D) Mr. Natwar Singh
(E) None of these
14.Noise pollution is measured in the unit called_____
(A) micron
(B) nautical miles
(C) ohms
(D) ampere
(E) decibel
15.Which of the following is not the name of popular IT/Software Company ?
(A) Wipro (B) Mastek
(C) Toyota (D) IBM
(E) Infosys

SBI Clerk interview Questions Asked

SBI Clerk interview Questions Asked
1)What is ATM ?
2)What are the functions of RBI?
3)Who will issue one rupee notes?
4)Who will issue currency notes?
5)what r the different types of shares?
6)what do u mean by CRR?
7)what is bank rate?
8)what do u mean by NPA????
9)What is micro finance ?
10)what is present share value of sbi?
11)What dou mean by Repo Rate ???
12)what is cost accounting ?
13)What are Accounting Concept?
14)What is Cost Accounting
15)what will u do if u have received a fake note from customer?
16)what is the difference between cheque and draft
17)what do you mean by Bank reconcilation statement
18)why should i select you?
19)what is relation between banks and stock market?

HCL Written test Paper c questions

HCL Written test Paper c questions
1. Which of the following about the following two declaration is true
i ) int *F()
ii) int (*F)()

Choice :
a) Both are identical
b) The first is a correct declaration and the second is wrong
c) The first declaraion is a function returning a pointer to an integer and the second is a pointer to function returning int
d) Both are different ways of declarin pointer to a function
Ans:c)
2. What are the values printed by the following program?
#define dprintf(expr) printf(#expr “=%d\n”,expr)
main()
{
int x=7;
int y=3;
dprintf(x/y);
}
Choice:
a) #2 = 2 b) expr=2 c) x/y=2 d) none
Ans : c)x/y=2
3. Which of the following is true of the following program
main()
{
char *c;
int *p;
c =(char *)malloc(100);
p=(int *)c;
free(p);
}
Ans : The code functions properly releasing all the memory allocated
4.output of the following.
main()
{
int i;
char *p;
i=0X89;
p=(char *)i;
p++;
printf(“%x %x\n”,p,i);
}
Ans : 0X8A
5. Which of the following is not a ANSI C language keyword?
Ans : Function
6. When an array is passed as parameter to a function, which of the following statement is correct choice:
a) The function can change values in the original array
b) In C parameters are passed by value. The funciton cannot change the original value in the array
c) It results in compilation error when the function tries to access the elements in the array
d) Results in a run time error when the funtion tries to access the elements in the array
Ans : a)
7. The type of the controlling expression of a switch statement cannot be of the type
a) int b) char c) short d)float e) none
Ans : d) float
8.What is the value of the expression (3^6) + (a^a)?
a) 3 b) 5 c) 6 d) a+18 e) None
Ans : 5
9. What is the value assigned to the variable X if b is 7 ?
X = b>8 ? b <<3 : b>4 ? b>>1:b;
a) 7 b) 28 c) 3 d) 14 e) None
Ans : 3;
10. Which is the output produced by the following program
main()
{
int n=2;
printf(“%d %d\n”, ++n, n*n);
}
a) 3,6 b) 3,4 c) 2,4 d) cannot determine
Ans : b) 3,4
11. What is th output of the following program?
int x= 0×65;
main()
{
char x;
printf(“%d\n”,x);
}
a) compilation error b) ‘A’ c) 65 d) unidentified
12. What is the output of the following program
main()
{
int a=10;
int b=6;
if(a=3)
b++;
printf(“%d %d\n”,a,b++);
}
a) 10,6 b)10,7 c) 3,6 d) 3,7 e) none
Ans : d) 3,7
13. What can be said of the following program?
main()
{
enum Months {JAN =1,FEB,MAR,APR};
Months X = JAN;
if(X==1)
{
printf(“Jan is the first month”);
}
}
a) Does not print anything
b) Prints : Jan is the first month
c) Generates compilation error
d) Results in runtime error
Ans : b)
14. What is the output of the following program?
main()
{
char *src = “Hello World”;
char dst[100];
strcpy(src,dst);
printf(“%s”,dst);
}
strcpy(char *dst,char *src)
{
while(*src) *dst++ = *src++;
}
a) “Hello World” b)”Hello” c)”World” d) NULL e) unidentified
Ans : d) NULL
15. What is the output of the following program?
main()
{
int l=6;
switch(l)
{ default : l+=2;
case 4: l=4;
case 5: l++;
break;
}
printf(“%d”,l);
}
a)8 b)6 c)5 d)4 e)none
Ans : c)5
16. What is the output of the following program?
main()
{
int x=20;
int y=10;
swap(x,y);
printf(“%d %d”,y,x+2);
}
swap(int x,int y)
{
int temp;
temp =x;
x=y;
y=temp;
}
a)10,20 b) 20,12 c) 22,10 d)10,22 e)none
Ans : d)10,22
17. What is the output of the f
ollowing problem ?

#define INC(X) X++
main()
{
int X=4;
printf(“%d”,INC(X++));
}
a)4 b)5 c)6 d)compilation error e) runtime error
Ans : d) compilation error
18. what can be said of the following
struct Node {
char *word;
int count;
struct Node left;
struct Node right;
}
a) Incorrect definition
b) structures cannot refer to other structure
c) Structures can refer to themselves. Hence the
statement is OK
d) Structures can refer to maximum of one other structure
Ans : c)
19. What is the size of the following union. Assume that the size of int =2, size of float =4 and size of char =1.
Union Tag{
int a;
flaot b;
char c;
};
a)2 b)4 c)1 d) 7
20. What is the output of the following program? (has been used to indicate a space)
main()
{
char s[]=”Hello,.world”;
printf(%15.10s”,s);
}
a)Hello,.World…
b)….Hello,.Wor
c)Hello,.Wor….
d)None of the above
(1) The combined length of the longer two pieces of rope is 12 metres.
(2) The combined length of the shorter two pieces of rope is 11 metres.
(A) (B) (C) (D) (E)
21. A certain company paid bonuses of Rs. 125 to each of its executive emplyees and Rs 75 to each of its nonexecutive employees. If 100 of the employees were nonexecutive, how
many were executive?

1) The company has a total of 120 employees
2) The total amount that the company paid in bonuses to its employees was Rs.10,000
(A) (B) (C) (D) (E)
22. What fraction of his salary did Mr. Johnson put into savings last week ?
1) Last week Mr.Johnson put Rs 17 into savings.
2) Last week Mr.Johnson put 5% of his salary into savings.
(A) (B) (C) (D) (E)
23. Each M-type memory unit will increase the base memory capacity of a certain computer by 3 megabytes. What is the base memory capacity, in megabytes, of the computer ?
1) 2 M-type memory units will increase the computer’s base memory capacity by 300%
2) The memory capacity of the computer after 2-M type memory units are added to the base memory capacity, is 1.6 times the memory capacity of the computer after 1 M-type unit is
added to the base memory capacity.
(A) (B) (C) (D) (E)
24. What fractional part of the total surface area of cube C is red?
1) Each of 3 faces of C is exactly 1/2 red
2) Each of 3 faces of C is entirely white
(A) (B) (C) (D) (E)
Section-C
Instructions ALL NUMBER USED ARE REAL NUMBERS FOLLOWING EACH QUESTIONS ARE FIVE POSSIBLE ANSWERS LABELED
A).B).C).D)&E).TICK THE BEST CHOICE.

14. How many of the integers between 25 and 45 are even ?
(A)21 (B)20 (C)11 (D)10 (E)9
Ans : d)10
15. If taxi fares were Rs 1.00 for the first 1/5 mile and Rs 0.20 for each 1/5 miles thereafter. The taxi fare for a 3-mile ride was
(A)Rs 1.56 (B)Rs 2.40 (C)RS 3.00 (D)Rs 3.80 (E)Rs 4.20
Ans : d)Rs 3.80
16. A computer routine was developed to generate two numbers (x, y) the first being a random number between 0 and 100 inclusive, and the second being less than or equal to the
square root of the first. Each of the following pair satisfies the routine EXCEPT

(A) (99.10) (B) (85.9) (C) (50.7) (D) (1.1) (E) (1.0)
Ans : A) (99.10)
17. A warehouse had a square floor with area 10,000 sq.meters. A rectangular addition was built along one entire side of the warehouse that increased the floor by one-half as much as the original floor. How many meters did the addition extend beyond the original buildings ?
(A)10 (B)20 (C)50 (D)200 (E)500
Ans : c) 50

Puzzles ebook Free Download

Puzzles ebook Free Download
A good collection of 300 Puzzles sutable for all kind of job interviews like
Google
Microsoft
Infosys...etc
Click here to download from www.cinterviews.com

Flex Interview Questions

Flex Interview Questions
1.Is it possible to make httpService Requests synchronous?
2.I need to load an image from flickr into my application. Do I need a crossdomain.xml file on flickr?
3.What is the difference between httpService and Data Service?
4.How do you generate random numbers within a given limit with actionscript?
5.Have you built any components with actionscript? If so explain how you did it?
6.How do you implement push on a flex applications?
7.I am going to add images into a tag. How will it resize itself?
8.I am going to add images into a tag. How will it resize itself?
9.What is a resource Manager?
10.What are the similarities between java and flex?
11.Why is the dynamic keyword used for in flex?
12.How do you implement push with flex data services?
13.What are the methods called when a UI component is intialized?
14.Can you write to the file system from flex?
15.What is a drag manager?
16.How do you call javascript from Flex?
17.How do you use a repeater in flex?
18.What are three ways to skin a component in flex?
19.How do you use css styles in flex?
20.What is the difference between sealed class and dynamic classes?

Drupal Interview Faqs part 2

Drupal Interview Faqs part 2
1.Explain Taxonomy in drupal?
2.How database system of drupal works?
3.Explain the path system of drupal?
4.Explain Region, Block, Menu in drupal?
5.How To Define New Regions in Drupal?
6.Explain the function and working of Dashboard module?
7.List the modules required for building a social networking website in Drupal?
8.List the SEO modules available in Drupal?
9.How to post videos from mobile to Drupal website?
10.List the features of Drupal?
11.How to port a joomla template to drupal?
12.Explain the capabilities of views module?
13.What are systems requirements for drupal installation?
14.What are the browser requirements for Drupal?
15.What is PDO?
16.How to enable clean urls in drupal?
17.Which are the core required modules in drupal 6.x ?
18.Is it possible to disable the core required modules through drupal admin?
19.Which are the core optional modules in drupal 6.x ?
20.What a module is in Drupal and what the process of writing one involves?
21.Drupal is flexible at handling events automatically and employing triggers. How do developers make use of these features?
22.The search features in Drupal are excellent, as compared to search in other content management systems. What makes these so good?

Drupal Interview Questions part 1

Drupal Interview Questions
1.What is Drupal?
2.What is content management system?CMS?
3.Which are commonly used PHP based CMS?
4.What is an Open source software ?
5.What are GNU Licenses ?
6.Why are so many Drupal versions available - 4.x, 5.x ...? Which one should I use?
7.Can I use Drupal on the command line?
8.What are hooks in Drupal?
9.What is Database abstraction layer in Drupal?
10.Explain the menu system in Drupal?Purpose of menus?
11.How to interact with Drupal search system?
12.What is a Module in drupal?
13.Explain User, Permission, Role in drupal?
14.Explain the concept of node in drupal?
15.Explain Concept of Comment in Drupal?

Accounting Interview Questions with Answers

Accounting Interview Questions with Answers
1)What do Debit and Credit terms mean to a financial accountant?
Answer:On 1st January 1998 the provision for Bad Debts stood at Rs.1, 100/-. During the year Bad Debts totaled to Rs.900/-. At the end of the year the Sundry Debtors were Rs.24, 000/-. The firm wishes to maintain the provision for Bad Debts @ 5% on Debts. In the year 1999 Bad Debts totaled to Rs.1, 600/- and at the end of the year the Debtors were Rs.42, 000/-. At the end of year 2000, the sundry Debtors were Rs.48, 500/- out of which Rs.500 were bad and had to be written off. In all the three years mentioned above firm was maintaining the provision @ 5% Debtors. You are required to choose the correct answer from the question from the options given below: (all working should a part of your answer). a) The provision for bad debts account at the end of 1998 shows a balance of : i) Rs.1, 100 ii) Rs.1, 200 iii) Rs.1, 155 iv) Rs.1, 400 b) The sundry debtors amount shown in the balance sheet of 31Dec., 1998 was ---- i) Rs.23, 100 ii) Rs.21, 900 iii) Rs.22, 800/- iv) Rs.21, 945 c) The amount of Bad Debts and provision for Bad Debts put together charged to the Profit and Loss A/c for the year ended 31st December 1999 was ---- i) Rs.2, 500 ii) Rs.3, 77 iii) Rs.2, 020 iv) Rs.3, 620 d) The provision for Bad Debts account at the end of the year 2000 shows a balance of ---- i) Rs.2, 400 ii) Rs.4, 000 iii) Rs.2, 425 iv) Rs.1, 900 e) The Sundry Debtors shown in the Balance Sheet of 31st December 2000 amounts to ---- i) Rs.48, 500 ii) Rs.48, 000 iii) Rs.45, 600 iv) Rs.46, 075. Ans to Q.No.2. a) ii) PBD – 5% of Rs.24, 000 = Rs.1, 200 b) iii) S.Debtors – Rs.24, 000 less Rs.1, 200 = Rs.22, 800 c) i) Rs.2, 500 (Bad debts Rs.1, 600 plus new PBD Rs.2, 100 minus old PBD Rs.1, 200) Note: Bad Debts are already written off in the year 1998 and 1999. d) i) Rs.2, 400 (Debtors Rs.48, 500 less Bad Debts Rs.500 and then 5% PBD) e) iii) Rs.45, 600 (Debtors Rs.48, 500 less Bad Debts Rs.500 and PBD Rs.2, 400).
2)What do Accrual Accounting System term mean to a financial accountant?
Answer:Accrual Accounting System: Accounting is normally done on accrual basis i.e. income and expenses are recorded as and when they become due and as and when they are actually received and paid. b. Debit and credit: Debit means an entry made on the left hand side of an account prepared in the horizontal format, an account which receive the benefit of a transaction. And credit made an entry made on the right hand side of an account prepared in the horizontal format, an account that gives the benefit of transaction. c. Transposition error: An error made in accounts while entering the total of debit to credit or vice-versa before calculating the balance or when wrongly an amount is carried forward to the next page of the same account. d. Capital Work in progress: The value of unfinished project or construction work as on date of balance sheet, whose work is still in progress. e. Statutory Reserves: Compulsory reserve to be maintained by an organization according to the statutes governed by it. f. Fluctuating capital method: Under this method, all the transaction relating to a partner are entered in only one capital account maintained for him. No current account is opened as in the Fixed Capital Method. Capital account is credited not only with the amount contributed by him as capital but other transactions such as interest on capital, drawings and share of profits are also recorded in the same capital account. g. Non-performing assets: Assets, which do not bring any revenue to the firm, is known as non-performing assets
3)Does the fixed capital of partners also change?
Answer:Yes. The fixed capital of partners also changes in the following two conditions:i. When a partner introduces additional amount as capital in the firm i.e. additional capital
ii. When a partner withdraws an amount of capital invested in the firm i.e. drawings against capital/drawings out of capital.
4)Is written partnership agreement necessary to form a partnership firm?
Answer:No. A partnership firm can be constituted with a partnership agreement implied between/among the partners while it is advisable only to have a written partnership agreement to avoid future disputes and also to produce to the income tax authorities and other interested parties dealing with the firm.
5)Give four items that may appear on the credit side of a Partner’s Current Account?
Answer:The four items that may appear on the credit side of a Partner’s Current Account are:
i.Salary payable to the Partner;
ii.Commission payable to the partner;
iii.Profit-share of the partner;
iv.Interest on Partner’s Capital.
6.What do you mean by past adjustments?
Answer:Past adjustments refer to the journal entries passed to adjust the effect of the transactions either recorded erroneously or omitted in the past. Past adjustments may also occur as a consequence of change in the terms of the partnership agreement with retrospective effect. For example; interest on drawings charged excessively, interest on capital omitted or treating manager as a partner with retrospective effect.
7.What is called average capital?
Answer:The weighted average with reference to time the partner’s capital has been used in the business is computed to determine the ratio in which profit or loss will be shared by the partners. This is called Average Capital.

MicroProcessor Interview Questions

MicroProcessor Interview Questions

What is a Microprocessor?
Microprocessor is a program-controlled device, which fetches the instructions from memory, decodes and executes the instructions. Most Micro Processor are single- chip devices.
What are the flags in 8086?
In 8086 Carry flag, Parity flag, Auxiliary carry flag, Zero flag, Overflow flag, Trace flag, Interrupt flag, Direction flag, and Sign flag.
Why crystal is a preferred clock source?
Because of high stability, large Q (Quality Factor) & the frequency that doesn?t drift with aging. Crystal is used as a clock source most of the times.
In 8085 which is called as High order / Low order Register?
Flag is called as Low order register & Accumulator is called as High order Register.
What is Tri-state logic?
Three Logic Levels are used and they are High, Low, High impedance state. The high and low are normal logic levels & high impedance state is electrical open circuit conditions. Tri-state logic has a third line called enable line.
What happens when HLT instruction is executed in processor?
The Micro Processor enters into Halt-State and the buses are tri-stated.
Which Stack is used in 8085?
LIFO (Last In First Out) stack is used in 8085.In this type of Stack the last stored information can be retrieved first
What is Program counter?
Program counter holds the address of either the first byte of the next instruction to be fetched for execution or the address of the next byte of a multi byte instruction, which has not been completely fetched. In both the cases it gets incremented automatically one by one as the instruction bytes get fetched. Also Program register keeps the address of the next instruction.
What are the various registers in 8085?
Accumulator register, Temporary register, Instruction register, Stack Pointer, Program Counter are the various registers in 8085
What is 1st / 2nd / 3rd / 4th generation processor?
The processor made of PMOS / NMOS / HMOS / HCMOS technology is called 1st / 2nd / 3rd / 4th generation processor, and it is made up of 4 / 8 / 16 / 32 bits.
Name the processor lines of two major manufacturers?
High-end: Intel - Pentium (II, III, 4), AMD - Athlon. Low-end: Intel - Celeron, AMD - Duron. 64-bit: Intel - Itanium 2, AMD - Opteron.
What?s the speed and device maximum specs for Firewire?
IEEE 1394 (Firewire) supports the maximum of 63 connected devices with speeds up to 400 Mbps. Where?s MBR located on the disk? Main Boot Record is located in sector 0, track 0, head 0, cylinder 0 of the primary active partition.
Where does CPU Enhanced mode originate from?
Intel?s 80386 was the first 32-bit processor, and since the company had to backward-support the 8086. All the modern Intel-based processors run in the Enhanced mode, capable of switching between Real mode (just like the real 8086) and Protected mode, which is the current mode of operation.
How many bit combinations are there in a byte?
Byte contains 8 combinations of bits.
Have you studied buses? What types?
There are three types of buses.
Address bus: This is used to carry the Address to the memory to fetch either Instruction or Data.
Data bus : This is used to carry the Data from the memory.
Control bus : This is used to carry the Control signals like RD/WR, Select etc.
What is the Maximum clock frequency in 8086?
5 Mhz is the Maximum clock frequency in 8086.
What is meant by Maskable interrupts?
An interrupt that can be turned off by the programmer is known as Maskable interrupt.
What is Non-Maskable interrupts?
An interrupt which can be never be turned off (ie. disabled) is known as Non-Maskable interrupt
What are the different functional units in 8086?
Bus Interface Unit and Execution unit, are the two different functional units in 8086.
What are the various segment registers in 8086?
Code, Data, Stack, Extra Segment registers in 8086.
What does EU do?
Execution Unit receives program instruction codes and data from BIU, executes these instructions and store the result in general registers.
Which Stack is used in 8086? k is used in 8086?
FIFO (First In First Out) stack is used in 8086.In this type of Stack the first stored information is retrieved first.
What are the flags in 8086?
In 8086 Carry flag, Parity flag, Auxiliary carry flag, Zero flag, Overflow flag, Trace flag, Interrupt flag, Direction flag, and Sign flag.
What is SIM and RIM instructions?
SIM is Set Interrupt Mask. Used to mask the hardware interrupts.
RIM is Read Interrupt Mask. Used to check whether the interrupt is Masked or not.
What is the difference between 8086 and 8088?
The BIU in 8088 is 8-bit data bus & 16- bit in 8086.Instruction queue is 4 byte long in 8088and 6 byte in 8086.
Give example for Non-Maskable interrupts?
Trap is known as Non-Maskable interrupts, which is used in emergency condition.
Give examples for Micro controller?
Z80, Intel MSC51 &96, Motorola are the best examples of Microcontroller.

CSC Fresher Placement Paper

CSC Fresher Placement Paper
Written Test
I.APTITUDE
In this round, they asked 40 questions in 40 minutes which includes
Two from Venn diagram, (easy)
One from probability,
3*3 Sudoku like below, (very easy)
8 2
9
You have to fill the numbers from 1-9 in the boxes, such that it should have 15 from top to bottom and across the diagonal, then you would have the box as like the one below:
8 2 5
1 4 7
6 9 3
From this you might have questions like, summing up the numbers which are right and left to the number 2. For us, three questions were asked from this:
*Some four questions were asked from the four different passages.
*One problem based on age. (easy)
*Three questions based on Speed and Distance.
*One from permutation
*Percentage ( easy )
*Few questions based on functions. (very easy one)
II. TECHNICAL

It consists of 75 questions and duration is 40 minutes. We had the questions from previous year papers.
1. ------- is associated with web services.
a) WSDL b) WML c) web sphere d) web logic
Ans: a
2.any large single block of data stored in a database, such as a picture or sound file, which does not include record fields, and cannot be directly searched by the database’s search engine.
a) TABLE
b) BLOB
c) VIEW
d) SCHEME
Ans: b
3.A reserved area of the immediate access memory used to increase the running speed of the computer program.
a) session memory
b) bubble memory
c) cache memory
d) shared memory
Ans: c
4.a small subnet that sit between a trusted internal network and an untruster external network, such as the public internet.
a) LAN
b) MAN
c) WAN
d) DMZ
Ans: d
5.technologies that use radio waves to automatically identify people or objects,which is very similar to the barcode identification systems,seen in retail stores everyday.
a) BLUETOOTH
b) RADAR
c) RSA SECURE ID
d) RFID
Ans: d
6.main(){
float fl = 10.5;
double dbl = 10.5
if(fl ==dbl)
printf(“UNITED WE STAND”);
else
printf(“DIVIDE AND RULE”)
}
What is the output?
a) compilation error
b) UNITED WE STAND
c) DIVIDE AND RULE
d) Linkage error.
Ans: c
7.main(){
static int ivar = 5;
printf(“%d”,ivar--);
if(ivar)
main();
}
What is the output?
a)1 2 3 4 5
b) 5 4 3 2 1
c)5
d) Compiler error:main cannot be recursive function.
Ans: b
8.main()
{
extern int iExtern;
iExtern = 20;
printf(“%d”,iExtern);
}
What is the output?
a)2
b) 20
c) compile error
d) linker error
Ans: d
9..#define clrscr() 100
main(){
clrscr();
printf(“%d\n\t”, clrscr());
}
What is the output?
a)100 b)10 c)compiler errord)linkage error
Ans: a
10.main()
{
void vpointer;
char cHar = ‘g’, *cHarpointer = “GOOGLE”;
int j = 40;
vpointer = &cHar;
printf(“%c”,*(char*)vpointer);
vpointer = &j;
printf(“%d”,*(int *)vpointer);
vpointer = cHarpointer;
printf(“%s”,(char*)vpointer +3);
}
What is the output?
a) g40GLE
b) g40GOOGLE
c) g0GLE
d) g4GOO
Ans: a
11.#define FALSE -1
#define TRUE 1
#define NULL 0
main() {
if(NULL)
puts(“NULL”);
else if(FALSE)
puts(“TRUE”);
else
puts(“FALSE”);
}
What is the output?
a) NULL
b) TRUE
c) FALSE
d)0
Ans: b
12.main() {
int i =5,j= 6, z;
printf(“%d”,i+++j);
}
What is the output?
a)13
b)12
c)11
d) Compiler error
Ans: c
13.main() {
int i ;
i = accumulator();
printf(“%d”,i);
}
accumulator(){
_AX =1000
}
What is output?
a)1
b)10
c)100
d)1000
Ans: d
14.main() {
int i =0;
while(+(+i--)!= 0)
i- = i++;
printf(“%d”,i);
}
What is the output?
a) -1
b) 0
c) 1
d) Will go in an infinite loop
Ans: a
15.main(){
int i =3;
for(; i++=0;)
printf((“%d”,i);
}
What is the output?
a) 1
b) 2
c) 1 2 3
d) Compiler error : L value required.
Ans: d
16. main(){
int i = 10, j =20;
j = i ,j?(i,j)?i :j:j;
printf(“%d%d”,i,j);
}
What is the output?
a) 20 20
b) 20 10
c) 10 20
d) 10 10
Ans: d
17.main(){
extern i;
printf(“%d\t”,i);{
int i =20;
printf(“%d\t”,i);
}
}
What is the output?
a) “Extern valueof i “ 20
b) Externvalue of i”
c) 20
d) linker Error: unresolved external symbol i
Ans: d
18.int DIMension(int array[]){
return sizeof(array/sizeof(int);}
main(){
int arr[10];
printf(“Array dimension is %d”,DIMension(arr));
}
What is output?
a) array dimension is 10
b) array dimension is 1
c) array dimension is 2
d) array dimension is 5
Ans: b
19. main(){
void swap();
int x = 45, y = 15;
swap(&x,&y);
printf(“x = %d y=%d”x,y);
}
void swap(int *a, int *b){
*a^=*b, *b^=*a, *a^ = *b;
What is the output?
a) x = 15, y =45
b) x =15, y =15
c) x =45 ,y =15
d) x =45 y = 45
Ans: a
20.main(){
int i =257;
int *iptr =&i;
printf(“%d%d”,*((char*)iptr),*((char *)iptr+1));
}
What is output?
a)1, 257
b)257 1c)0 0d)1 1
Ans: d
21.main(){
int i =300;
char *ptr = &i;
*++ptr=2;
printf(“%d”,i);
}
What is output?
a) 556
b) 300
c) 2
d) 302
Ans: a
22.#include
main(){
char *str =”yahoo”;
char *ptr =str;
char least =127;
while(*ptr++)
least = (*ptr
printf(“%d”,least);
}
What is the output?
a) 0
b)127
c) yahoo
d) y
Ans: a
23.Declare an array of M pointers to functions returing pointers to functions returing pointers to characters.
a) (*ptr[M]()(char*(*)());
b) (char*(*)())(*ptr[M])()
c) (char*(*)(*ptr[M]())(*ptr[M]()
d) (char*(*)(char*()))(*ptr[M])();
24.void main(){
int I =10, j=2;
int *ip = &I ,*jp =&j;
int k = *ip/*jp;
printf(“%d”,k);
}
What is the output?
a) 2
b) 5
c) 10
d) compile error:unexpected end of file in comment started in line 4
Ans: d
25.main(){
char a[4] =”GOOGLE”;
printf(“%s”,a);
}
What is the output?
a) 2
b) GOOGLE
c) compile error: yoo mant initializers
d) linkage error.
Ans: c
26.For 1MB memory, the number of address lines required
a) 12
b) 16
c) 20
d) 32
Ans: 16
27.There is a circuit using 3 nand gates with 2 inputes and 1 output,f ind the output.
a) AND
b) OR
c) XOR
d) NAND
Ans: b (not sure)
28. What is done for push operation
a) SP is incremented and then the value is stored.
b) PC is incremented and then the value is stored.
c) PC is decremented and then the value is stored.
d) SP is decremented and then the value is stored.
Ans: d
29.Memory allocation of variables declared in a program is:
a) Allocated in RAM
b) Allocated in ROM
c) Allocated in stack
d) Assigned in registers.
Ans: c
30.What action is taken when the processer under execution is interrupted by TRAP in 8085MPU?
a) Processor serves the interrupt request after completing the execution of the current instruction.
b) processer serves the interrupt request after completing the current task.
c) processor serves the interrupt immediately.
d) processor serving the interrupt request depent deprnds upon the priority of the current task under execution.
Ans: a
31.purpose of PC (program counter)in a microprocessor is:
a) To store address of TOS(top of stack)
b) To store address of next instructions to be executed
c) count the number of instructions
d) to store the base address of the stack.
Ans: b
32.conditional results after execution of an instruction in a microprocess is stored in
a) register
b) accumulator
c) flag register
d) flag register part of PSW (program status word)
Ans: c
33.The OR gate can be converted to the NAND function by adding----gate(s)to the input of the OR gate.
a) NOT
b) AND
c) NOR
d) XOR
Ans: a
34. In 8051 microcontroller , has a dual function.
a) port 3
b) port 2
c) port 1
d) port 0
Ans: b
35.An 8085 based microprocessor with 2MHz clock frequency,will execute the following chunk of code with how much delay?
MVI B,38H
HAPPY: MVI C, FFH
SADDY: DCR C
JNZ SADDY
DCR B
JNC HAPPY
a) 102.3
b)114.5
c)100.5
d)120
36.In 8085 MPU what will be the status of the flag after the execution of the following chunk of code.
MVI B,FFH
MOV A,B
CMA
HLT
a) S = 1, Z = 0, CY = 1
b) S = 0, Z = 1, CY = 0
c) S = 1, Z = 0, CY = 0
d) S = 1, Z = 1 ,CY = 1
37.A positive going pulse which is always generated when 8085 MPU begins the machine cycle.
a) RD
b) ALE address latch enable…
c) WR
d) HOLD
Ans: b
38.when a ----- instruction of 8085 MPU is fetched , its second and third bytes are placed in the W and Z registers.
a) JMP
b) STA
c) CALL
d) XCHG
Ans: c
39.what is defined as one subdivision of the operation performed in one clock period.
a) T- State
b) Instruction Cycle
c) Machine Cycle
d) All of the above
Ans: a
40.At the end of the following code, what is the status of the flags.
LXI B, AEC4H
MOV A,C
ADD HLT
a) S = 1, CY = 0, P = 0 , AC = 1
b) S =0 , CY = 1, P = 0,AC = 1
c) S = 0, CY = 1, P = 0 , AC = 1
d) S = 0, CY = 1, P = 1 , AC = 1
41.In 8051 micro controller what is the HEX number in the accumulator after the execution of the following code.
MOV A,#0A5H
CLR C
RRC A
RRC A
RL A
RL A
SWAP A
a)A6
b)6A
c)95
d)A5.
Ans: a
42.The Pentium processor requires ------------ volts.
a)9 b)12 c)5 d)24
ans; c
43. The data bus on the Celeron processor is-------bits wide.
a)64 b)32 c)16 d)128. Ans: a
44.K6 processor
a) Hitachi b) toshiba c) zilog d) AMD. Ans: d
45. What is the control word for 8255 PPI,in BSR mode to set bit PC3.
a)0EH b)0FH c)07H d)06H. ans:c
46.The repeated execution of a loop of code while waiting for an event to occur is called ---------.The cpu is not engaged in any real productive activity during this period,and the process doesn’t progress towards completion.
a) dead lock b) busy waiting c) trap door d) none.
Ans: b
47. Transparent DBMS is defined as
a) A DBMS in which there are no program or user access languages. b) A DBMS which has no cross file capabilities but is user friendly and provides user interface management. c) A DBMS which keeps its physical structure hidden from user d) none.
Ans: c
48.Either all actions are carried out or none are. users should not have to worry about the effect of incomplete transctions.DBMS ensures this by undoing the actions of incomplete transctions.this property is known as
a) Aggregation b) atomicity c) association d) data integrity.
ans : B…
49.------ algorithms determines where in available to load a program. common methods are first fit,next fit,best fit.--------- algorithm are used when memory is full , and one process (or part of a process) needs to be swaped out to accommodate a new program.The ------------- algorithm determines which are the partions to be swaped out.
a) placement, placement, replacement
b) replacement, placement, placement
c) replacement, placement, replacement
d) placement, replacement, replacement Ans: D
50.Trap door is a secret undocumented entry point into a program used to grant access without normal methods of access authentication. A trap is a software interrupt,usually the result of an error condition.
a)true b)false.
Ans: A
51. Given a binary search tree,print out the nodes of the tree according t5o post order traversal.
4
2 5
1 3
a)3,2,1,5,4. b)1,2,3,4,5. c)1,3,2,5,4. d)5,3,1,2,4. Ans: C
52.which one of the following is the recursive travel technique.
a)depth first search b)preorder c)breadth first search d)none.
53.which of the following needs the requirement to be a binary search tree.
a) 5
/ \
2 7
/
1
b) 5
/ \
6 7
c) 5
/ \
2 7
/\
1 6
d) none.
54.in recursive implementations which of the following is true for saving the state of the steps
a) as full state on the stack
b) as reversible action on the stack
c) both a and b
d) none
55.which of the following involves context switch
a)previliged instruction
b)floating point exception
c)system calls
d)all
e)none
ans : c
56.piggy backing is a technique for
a)acknowledge
b)sequence
c)flow control
d)retransmission
ans:A
57. a functional dependency XY is ___________dependency if removal of any attribute A from X means that the dependency does not hold any more
a)full functional
b) multi valued
c)single valued
d)none
ans : a
58)a relation schema R is in BCNF if it is in ___________and satisfies an additional constraints that for every functional dependency XY,X must be a candidate key
a)1 NF
b)2 NF
c)3 NF
d)5 NF
59) a _________sub query can be easily identified if it contains any references to the parent sub query columns in the _________ clause
A) correlated ,WHERE
b) nested ,SELECT
c) correlated,SELECT
d) none
60) hybrid devise that combines the features of both bridge and router is known as
a)router b)bridge c)hub d)brouter
61) which of the following is the most crucial phase of SDLC
a)testing b)code generation c) analysys and design d)implementation
Ans: c
62)to send a data packet using datagram ,connection will be established
a)no connection is required
b) connection is not established before data transmission
c)before data transmission
d)none
Ans: c
63)a software that allows a personal computer to pretend as as computer terminal is
a) terminal adapter
b)terminal emulation
c)modem
d)none
Ans: b
64) super key is
a) same as primary key
b) primary key and attribute
c) same as foreign key
d) foreign key and attribute
Ans: b
65.In binary search tree which traversal is used for ascending order values
a) Inorder b)preorder c)post order d)none
Ans: a
66.You are creating an index on ROLLNO colume in the STUDENT table.which statement will you use?
a) CREATE INDEX roll_idx ON student, rollno;
b) CREATE INDEX roll_idx FOR student, rollno;
c) CREATE INDEX roll_idx ON student( rollno);
d) CREATE INDEX roll_idx INDEX ON student (rollno);
Ans: c
67.A________class is a class that represents a data structure that stores a number of data objects
a. container b.component c.base d.derived
Ans: a
68.Which one of the following phases belongs to the compiler Back-end.
a. Lexical Analysis b.Syntax Analysis c. Optimization d.Intermediate Representation.
Ans: c
69.Every context _sensitive language is context_free
a. true b.false
Ans: b
70.Input:A is non-empty list of numbers L
Xß-infinity
For each item in the list L,do
If the item>x,then
Xß the item
Return X
X represents:-
a)largest number
b)smallest number
c)smallest negative number
d) none
71.Let A and B be nodes of a heap,such that B is a child of A. the heap must then satisfy the following conditions
a)key(A)>=key(B)
b)key(A)
c)key(A)=key(B)
d)none
72.String ,List,Stack,queue are examples of___________
a)primitive data type
b)simple data type
c)Abstract data type
d)none
Ans: c
73.which of the following is not true for LinkedLists?
a)The simplest kind of linked list is a single linked list ,which has one link per node .this link points to the next node in the list,or to a null value or emptylist if it is the last node.
b)a more sophisticated kind of linked list is a double linkedlist or two way linkedlist .Each node has two links ,one to the previous node and one to the next node.
c) in a circleLinkedList ,the first and last nodes are linked together.this can be done only for double linked list.
d) to traverse a circular linkedlist ,u begin at any node and follow the list in either direction until u return to the original node.
Ans: c
74.sentinel node at the beginning and /or at the end of the linkedlist is not used to store the data
a) true
b) false
Ans:a

Followed by
2. GROUP DISCUSSION
3. TECHNICAL HR
4. PERSONAL INTERVIEW

New QTP Interview Questions and Answers

New QTP Interview Questions and Answers
1)Tell me the features and importance of Quick test professional(QTP)?
1. Key word driven testing
2. Suitable to web applications
3. Vb script as the script language
4. Better error handling mechanism
5. Excellent data driven testing features
6. Record and playback
7. to store a values we can use runtime data table
8. Auto documentation
2)Can a QTP user can increase or decrease active screen information with the test?
Yes, user can store.
3)What are the limitations of QTP?
The limitations listed below are specifically for QTP 8.2:
Maximum length of name—255
Maximum length of format string—255
Maximum number of tables (workbooks)—Limited by system resources (windows and memory)
Column width—0 to 255 characters
Text length—16,383 characters
Formula length—1024 characters
Number precision—15 digits
Largest positive number—9.99999999999999E307
Largest negative number— -9.99999999999999E307
Smallest positive number—1E-307
Smallest negative number— -1E-307
Maximum number of names per workbook—Limited by available memory
Maximum worksheet size—65,536 rows by 256 columns
4)What are the QTP Script Coding Standards?
at the start In Header section of QTP you have to declare these items
***Header Section of the QTP you have to Mention*****
1.Name of the Script
2.Name of the Test case
3.Name of the Reusable scripts
4.Path of the Excel sheet
5.Version
6.Name of the Tester
7.Change History
*******************************
Then you have to follow Naming convention for each of the script that you will create in QTP.Like Y_TRA_AGIALANT_FINANCE_01
*******************************
You have to follow coding standards for the variable that you will declare in the QTP script
Like For all the
Integer type variable the name should start with int
Character type variable the name should start with Char
Boolean type variable the name should start with Boo
String Type variable the name should start with Str
*****************************************************
Then throughout the QTP script you have to mention that what you are doing actually. Like
**Here you have to mention in the script what you are doing ** Like
**Clicking the yahoo signin button to Login **********
Browswer(Yahoo page).Page(Yahoo_1).Button(Sign_in).Click
*************************************************
Coding standard are used for readability and better understanding.

Testplan Interview Questions and answers

Testplan Interview Questions and answers
1)List the key elements of Test Plan? (or) What test plan should contains?
The key elements for a test plan are:
1.The requirement documents based on which the plan is developed. Like BRD,FRD
2.Testing environment
3.Test data
2)What is test strategy?
1. It Describe How to Test
2. It Define the methodology quality analyst finalizes test Responsibility Matrix on the basis of testing policy and this is done by QA.
3)Define test Plan?
Test Lead or project lead develop test plan. this phase starts after the SRS and other business knowledge gathering. Test Plan describes what to test1.What is Scope of test
2.What is Approach to test.
3.What are the modules to test.
4)What is the difference between Master Test Plan and Test Plan?
Test Plan: Test plan we write for each module (Both for black box and white box testing).It contains test cases,strategy that is being used and any stubs that are involved to test that particular module.
Master Test Plan : The MTP is required in all projects. The MTP is applicable for all the projects. It have details about all modules testing , the time frame, the resource, hardware dependencies, and any training required. It also involves a block diagram showing how the testing takes place.
5)What are Standards for Software Test Plans?
There is number of standards for test plans, IEEE also defines the test plans.
IEEE standards for test plans:
829-1983 IEEE Standard for Software Test Documentation
1008-1987 IEEE Standard for Software Unit Testing
1012-1986 IEEE Standard for Software Verification & Validation Plans
1059-1993 IEEE Guide for Software Verification & Validation Plans

Sample Unit Testing Interview Questions

Sample Unit Testing Interview Questions
1)What would be the Test Objective for Unit Testing? What are the quality measurements to assure that unit testing is complete?
2)What is use of NUnit for unit testing?
3)How to do unit testing with Visual studio team system?
4)What are the Phases of Unit Testing?

Unit Testing Interview Questions and Answers

Unit Testing Interview Questions and Answers
1)What is Unit Testing?
A.Testing of individual component of software.
2)At what phase of SDLC, Unit Testing Starts?
A.Unit testing starts with coding phase simultaneously after design phase.
Actually, Unit testing is performed by Developers. And it is done, once they are done with the completion of their code development. Like whenever code is done for a module or a project the unit testing starts, even you can do unit testing of a page also.
3)How you prepare Test data in unit testing process???
A.Preparing proper test data is a core part of any testing process. a developer or tester can develop a test data as per project requirement. Either it can be a production data or any dummy data. It is responsibility of a tester or developer to prepare good data for unit testing process. Tester should create his/her own test data additional to the existing standard production data. Your test data set should be ideal in terms of cost and time. Design our test data considering following categories:
Test data set examples:
1)No data: Run your unit test on blank or default data. and check if proper error messages are generated.
2)Valid data set: Create it to check if application is
functioning as per requirements and a valid input data is
properly saved in database or files with out any error.
3)Invalid data set: Prepare invalid data set to check
application behavior for negative values, alphanumeric
string inputs (Negative unit testing).
4)Illegal data format: Make one data set of illegal data
format. System should not accept data in invalid or illegal
format. Also check proper error messages are generated.
5)Boundary Condition data set: Check Data if it is out of range. Identify application boundary cases and prepare
data set that will cover lower as well as upper boundary
conditions.
6)Data set for performance, load and stress testing: This
data set should be large in volume.
This way creating separate data sets for each test
condition will ensure complete test coverage.

Project Management Testing Interview Questions

Project Management Testing Interview Questions
1)Tell me something about your previous job and organization?
2)Tell me some of the projects that you handled in the previous job?
3)How do you handle Stress in office?
4)Tell me some factors which motivates you?
5)What is MPMM?
6)How do you judge technical Skills of a tester/Developer?
7)How do you distribute task among the team members?
8)What are the features of project management tools?
9)As a project manager , how to add value to project?
10)What process will you follow for timely delivery of project?
11)What is importance of project planning?
12)How will you handle a less productive team member?
13)How to judge a project is successful?
14)What is project management methodology?
15)What tools and techniques you use in project management?
16)How to maintain quality in project with fast development?
17)How to implement Agile methodology in project?

Fresher Interview puzzles

Fresher Interview puzzles
1)There is a 50m long army platoon marching ahead. The last person in the platoon wants to give a letter to the first person leading the platoon. So while the platoon is marching he runs ahead, reaches the first person and hands over the letter to him and without stopping he runs and comes back to his original position. In the mean time the whole platoon has moved ahead by 50m. The question is how much distance did the last person cover in that time. Assuming that he ran the whole distance with uniform speed.
Answer
The last person covered 120.71 meters.
It is given that the platoon and the last person moved with uniform speed. Also, they both moved for the identical amount of time. Hence, the ratio of the distance they covered - while person moving forward and backword - are equal.
Let's assume that when the last person reached the first person, the platoon moved X meters forward.
Thus, while moving forward the last person moved (50+X) meters whereas the platoon moved X meters.
Similarly, while moving back the last person moved [50-(50-X)] X meters Whereas the platoon moved (50-X) meters.
Now, as the ratios are equal,
(50+X)/X = X/(50-X)
(50+X)*(50-X) = X*X
Solving, X=35.355 meters
Thus, total distance covered by the last person
= (50+X) + X
= 2*X + 50
= 2*(35.355) + 50
= 120.71 meters
Note that at first glance, one might think that the total distance covered by the last person is 100 meters, as he ran the total lenght of the platoon (50 meters) twice. TRUE, but that's the relative distance covered by the last person i.e. assuming that the platoon is stationary.

Puzzles on Daughters Age

Puzzles on Daughters Age
Two MIT math grads bump into each other at Fairway on the upper west side. They haven’t seen each other in over 20 years.
The first grad says to the second: “how have you been?”
second:“great! i got married and i have three daughters now”
first:“really? how old are they?”
second:“well, the product of their ages is 72, and the sum of their ages is the same as the number on that building over there..”
first:“right, ok.. oh wait.. hmm, i still don’t know”
second:“oh sorry, the oldest one just started to play the piano”
first:“wonderful! my oldest is the same age!”
problem:how old are the daughters?
Answer:start with what you know. you know there are 3 daughters whose ages multiply to 72. let’s look at the possibilities…
Ages: Sum of ages:
1 1 72 74
1 2 36 39
1 3 24 28
1 4 18 23
1 6 12 19
1 8 9 18
2 2 18 22
2 3 12 17
2 4 9 15
2 6 6 14
3 3 8 14
3 4 6 13
after looking at the building number the man still can’t figure out what their ages are (we’re assuming since he’s an MIT math grad, he can factor 72 and add up the sums), so the building number must be 14, since that is the only sum that has more than one possibility.

finally the man discovers that there is an oldest daughter. that rules out the “2 6 6” possibility since the two oldest would be twins. therefore, the daughters ages must be “3 3 8”.

(caveat: an astute reader pointed out that it is possible for two siblings to have the same age but not be twins, for instance one is born in january, and the next is conceived right away and delivered in october. next october both siblings will be one year old. if a candidate points this out, extra credit points to him/her.)

this question is pretty neat, although there is certainly a bit of an aha factor to it. the clues are given in such a way that you think you are missing information (the building number), but whats important isn’t the building number, but the fact that the first man thought that it was enough information, but actually wasn’t.

even if the candidate doesn’t know the solution, they could come up with some interesting thoughts. if they just stare at you and shrug “i dunno” then thank them for their time and don’t give them a fogcreek pen.

Tech Puzzle int atoi( char* pStr )

Tech Puzzle int atoi( char* pStr )
Question:Write the definition for this function without using any built-in functions. if pStr is null, return 0. if pStr contains non-numeric characters, either return 0 (ok) or return the number derived so far (better) (e.g. if its “123A”, then return 123). assume all numbers are positive. plus or minus signs can be considered non-numeric characters. in order to solve this program, the programmer must understand the difference between the integer 0 and the character ‘0’, and how converting ‘0’ to an int, will not result in 0. in other words, they have to understand what ascii is all about.
Answer:
string manipulation functions are great programming questions. they test whether the user can understand and translate into code simple algorithms. string functions test pointer arithmetic which usually shows a knowledgeable programmer. also there are usually multiple solutions, some more efficient than others. plus people use them all the time so they should understand how they work. my favorite is atoi and i start the problem like this:
int atoi( char* pStr )
write the definition for this function without using any built-in functions. if pStr is null, return 0. if pStr contains non-numeric characters, either return 0 (ok) or return the number derived so far (better) (e.g. if its “123A”, then return 123). assume all numbers are positive. plus or minus signs can be considered non-numeric characters. in order to solve this program, the programmer must understand the difference between the integer 0 and the character ‘0’, and how converting ‘0’ to an int, will not result in 0. in other words, they have to understand what ascii is all about. if they are stuck solving this problem, just ask them first to write:
charToInt(char c)
if they can’t do that then they basically missed half the problem. any moderately talented programmer who has a CS degree knows how to convert a char to an int. (note i said convert, not cast. charToInt('9') should return 9.)
when they start to solve the problem you will notice that they must make a choice in how they will process the string - from left to right or right to left. i will discuss both methods and the difficulties encountered in each.
“right to left” - this method starts at the right hand letter of the string and converts that character to an int. it then stores this value after promoting it to its correct “tens” place.
int atoi( char* pStr )
{
int iRetVal = 0;
int iTens = 1;
if ( pStr )
{
char* pCur = pStr;
while (*pCur)
pCur++;
pCur--;
while ( pCur >= pStr && *pCur <= '9' && *pCur >= '0' )
{
iRetVal += ((*pCur - '0') * iTens);
pCur--;
iTens *= 10;
}
}
return iRetVal;
}
“left to right” - this method keeps adding the number and multiplying the result by ten before continuing to the next number. e.g. if you had “6234” and you processed from left to right you’d have 6, then if you kept reading you’d multiply your result by 10 (6*10) to add a zero for where the next number would go. 60, and then you’d slide the 2 into the zero place you just made. 62. do it again, 620, slide the next number in, 623.
int atoi( char* pStr )
{
int iRetVal = 0;
if ( pStr )
{
while ( *pStr && *pStr <= '9' && *pStr >= '0' )
{
iRetVal = (iRetVal * 10) + (*pStr - '0');
pStr++;
}
}
return iRetVal;
}
i think the “left to right” method is a little bit cleaner, or maybe its just cooler. but both are “correct”.
remember that debugging code on paper is somewhat hard. most programmers aren’t used to studying code that much when you can just hit F-7, compile and see if the compiler barfs or not. if you notice an error, just ask them to step through a sample string drawing out what is happening with all the variables and the pointers in every step. they should find their mistake then and fix it (no points deducted).

Train Puzzle

Train Puzzle
Problem:two trains enter a tunnel 200 miles long (yeah, its a big tunnel) travelling at 100 mph at the same time from opposite directions. as soon as they enter the tunnel a supersonic bee flying at 1000 mph starts from one train and heads toward the other one. as soon as it reaches the other one it turns around and heads back toward the first, going back and forth between the trains until the trains collide in a fiery explosion in the middle of the tunnel (the bee survives). how far did the bee travel?
Answer:This puzzle falls pretty high on my aha scale. my first inclination when i heard it was to think “ok, so i just need to sum up the distances that the bee travels…” but then you quickly realize that its a difficult (not impossible) summation which the interviewer could hardly expect you to answer (unless i guess if you are looking for a job as a quant). “there must be a trick” you say. eh, sort of i guess, enough to say that this question is a stupid interview question.

the tunnel is 200 miles long. the trains meet in the middle travelling at 100 mph, so it takes them an hour to reach the middle. the bee is travelling 1000 mph for an hour (since its flying the whole time the trains are racing toward one another) - so basically the bee goes 1000 miles.

there is no process to explain, so this question can’t possibly teach you anything about the person. they either know it or they don’t and if they already knew it before you asked, you’re not going to be able to tell when they give you the answer. so don’t ask this question. and if someone asks you this question, just tell them you’ve already heard it before.

Reverse String Technics and Solutions

Reverse String Technics and Solutions
A typical programming interview question is “reverse a string, in place”. if you understand pointers, the solution is simple. even if you don’t, it can be accomplished using array indices. i usually ask candidates this question first, so they get the algorithm in their head. then i play dirty by asking them to reverse the string word by word, in place. for example if our string is “the house is blue”, the return value would be “blue is house the”. the words are reversed, but the letters are still in order (within the word).
Answer
Solving the initial problem of just reversing a string can either be a huge help or a frustrating hinderance. most likely the first attempt will be to solve it the same way, by swapping letters at the front of the string with letters at the back, and then adding some logic to keep the words in order. this attempt will lead to confusion pretty quickly.

for example, if we start by figuring out that “the” is 3 letters long and then try to put the “t” from “the” where the “l” from “blue” is, we encounter a problem. where do we put the “l” from “blue”? hmm… well we could have also figured out how long “blue” was and that would tell us where to put the “l” at… but the “e” from “blue” needs to go into the space after “the”. argh. its getting quite confusing. in fact, i would be delighted to even see a solution to this problem using this attack method. i don’t think its impossible, but i think it is so complex that it’s not worth pursuing.
here’s a hint. remember before when we just reversed “the house is blue”? what happened?
initial:the house is blue
reverse:eulb si esuoh eht
look at the result for a minute. notice anything? if you still don’t see it, try this.
initial:the house is blue
reverse:eulb si esuoh eht
wanted:blue is house the
the solution can be attained by first reversing the string normally, and then just reversing each word.

Puzzles: Ant Problem Question

Puzzles: Ant Problem Question
Question:Three ants are sitting at the three corners of an equilateral triangle. Each ant starts randomly picks a direction and starts to move along the edge of the triangle. What is the probability that none of the ants collide?
Answer:So let’s think this through. The ants can only avoid a collision if they all decide to move in the same direction (either clockwise or anti-clockwise). If the ants do not pick the same direction, there will definitely be a collision. Each ant has the option to either move clockwise or anti-clockwise. There is a one in two chance that an ant decides to pick a particular direction. Using simple probability calculations, we can determine the probability of no collision.
P(No collision) = P(All ants go in a clockwise direction) + P( All ants go in an anti-clockwise direction) = 0.5 * 0.5 * 0.5 + 0.5 * 0.5 * 0.5 = 0.25

Puzzle :Chasing dogs Question

Puzzle :Chasing dogs Question
Question:There are four dogs each at the corner of a unit square. Each of the dogs starts chasing the dog in the clockwise direction. They all run at the same speed and continuously change their direction accordingly so that they are always heading straight towards the other dog. How long does it take for the dogs to catch each other and where?
Answer:Let’s the dogs be A, B, C and D where A is chasing B, B is chasing C, C is chasing D and D is chasing A.
All the dogs will eventually meet in the center of the square. Since all the dogs move in symmetry, the only logical answer to the location of their meeting is the center of the square.
At any point in time, dog A is perpendicular to dog B and B perpendicular to C and so on. Dog A moves towards dog B but dog B does not move towards or away from dog A since it is moving perpendicular to dog A. Therefore, the distance that dog A needs to cover to reach dog B is the same as the original distance between them, one unit.

The speed of each of the dog towards the dog it is chasing is given by (1 + cos(t)) where t is the angle on each corner of the square.

Speed of dog = 1 + cos(90) = 1 + 0 = 1
Time needed = Distance/Speed = 1 / 1 = 1 unit.
Have a better solution? Let us know through the comments section!

Puzzle :Red Blue Marbles

Puzzle :Red Blue Marbles
Question:You have 50 red marbles, 50 blue marbles and 2 jars. One of the jars is chosen at random and then one marble will be chosen from that jar at random. How would you maximize the chance of drawing a red marble? What is the probability of doing so? All 100 marbles should be placed in the jars.
Answer:Seems tricky at first right? Given that the number of red and blue marbles are the same, you would tend to think that the odds are 50-50. You would try different combinations, such as 25 of each colored marble in a jar or putting all red marbles in one jar and all the blue in the other. You would still end up with a chance of 50%.

So lets think of a better way to distribute the marbles. What if you put a single red marble in one jar and the rest of the marbles in the other jar? This way, you are guaranteed at least a 50% chance of getting a red marble (since one marble picked at random, doesn’t leave any room for choice). Now that you have 49 red marbles left in the other jar, you have a nearly even chance of picking a red marble (49 out of 99).
So let’s calculate the total probability.
P( red marble ) = P( Jar 1 ) * P( red marble in Jar 1 ) + P( Jar 2 ) * P( red marble in Jar 2 )
P( red marble ) = 0.5 * 1 + 0.5 * 49/99
P( red marble ) = 0.7474

Thus, we end up with ~75% chance of picking a red marble.
Have a better solution? Let us know through the comments section!

Puzzle :Boxes of Money

Puzzle :Boxes of Money
Question:You are given b boxes and n dollar bills. The money has to be sealed in the b boxes in a way such that without thereafter opening a box, you can give someone a requested whole amount of dollars from 0 to n. How should b be related to n for this to happen?
Answer:Stumped? Let’s think of an example to approach this problem.
Say we have $100. A good approach to distributing $100 would be the binary number system. So you’d have $1, $2, $4, $8, $16, $32 in the first six boxes. We can’t fill the next box with $64 dollars because we are only left with $37 dollars (from a total of $100). So we’d have to put $37 in the seventh box. To supply any requested amount, we’d have to use a combination of these boxes.
To find out the restrictions on the values of b and n, we have to think of different scenarios. For instance, with a million dollars and just one box, we would never be able to dispense any requested amount less than a million. However, if we are ever in a situation with more boxes than dollars, there is a never a problem.
Using this approach, we can create a table showing the best relationship between b and n
b = 1 n = up to $1
b = 2 n = up to $2 + $1 = $3
b = 3 n = up to $4 + $2 + $1 = $7
b = 4 n = up to $8 + $4 + $2 + $1 = $15
See a pattern yet? So the best way we would be able to dispense any requested amount is to have n <= 2^b – 1.

Puzzles: Card Pariology Question

Puzzles: Card Pariology Question
Question: John and Matt decide to play a game using a deck of cards. The game involves flipping two cards at a time. If both cards are black, they go into John’s pile and if both cards are red, they go into Matt’s pile. If one card is red and one card is black, they go into a discard pile. The game is played until all 52 cards have been flipped. The person with the most cards in their pile wins. If there is a tie, John wins. If Matt has more cards than John, then John pays Matt a dollar. What is the chance of Matt getting a dollar?
Answer:Any guesses? The chance of Matt getting any money is zero. Do you know why?
Say the deck is arranged in a way such that there are 13 black pairs. In this situation, John gets 13 black pairs. And since there are no other black cards left, Matt gets 13 red pairs too. So there’s a tie and John wins.
Suppose there are 12 black pairs in the deck. In this case, there would be 2 black cards left in the deck that would pair up with 2 other red cards in the deck. These 2 black cards would never be together since we have already claimed that there are only 12 black pairs in the deck. As a result, 2 pairs of cards end up in the discard pile. The remaining cards would all be red and Matt would get 12 red pairs too. So once again, there’s a tie and John wins.
We could continue this process through induction. Assuming there are 11 black pairs, there would be 4 other black cards that pair up with 4 other red cards and go into the discard pile. Once again, both John and Matt end up with 11 pairs of cards each and John wins.
Here’s a table to explain each case.
John Discarded Matt
13 pairs 0 pairs 13 pairs
12 pairs 2 pairs 12 pairs
11 pairs 4 pairs 11 pairs
10 pairs 6 pairs 10 pairs
9 pairs 8 pairs 9 pairs
8 pairs 10 pairs 8 pairs
7 pairs 12 pairs 7 pairs
6 pairs 14 pairs 6 pairs
5 pairs 16 pairs 5 pairs
4 pairs 18 pairs 4 pairs
3 pairs 20 pairs 3 pairs
2 pairs 22 pairs 2 pairs
1 pair 24 pairs 1 pair
0 pairs 26 pairs 0 pairs

So we see that in any case, there is a tie between John and Matt. So Matt never ends up with a dollar.

CAT Exam Sample paper with solutions

CAT Sample paper with solutions
I Section
This section contains 25 questions

1.The price of Darjeeling tea (in rupees per kilogram) is 100 + 0.10n, on the nth day of2007 (n = 1, 2, …, 100), and then remains constant. On the other hand, the price of
Ooty tea (in rupees per kilogram) is 89 + 0.15n, on the nth day of 2007 (n = 1, 2, …,
365). On which date in 2007 will the prices of these two varieties of tea be equal?
(1) May 21
(2) April 11
(3) May 20
(4) April 10
(5) June 30
Solution:
Note that the price of Darjeeling tea remains constant after the 100th day (n = 100).
If the prices of the two varieties of tea become equal before n = 100, then
100 + 0.1n = 89 + 0.15n
n = 220, which is not possible. (Since n is assumed to be less than 100)
The prices of the two varieties will be equal after n = 100,
i.e., when the price of Darjeeling tea = 100 + 0.1 × 100 = 110
89 + 0.15n = 110
n = 140
2007 is not a leap year. Number of days till 30th April = 31 + 28 + 31 + 30 = 120
The prices of the two varieties will be equal on 20th May.
Hence, option 3.
2.A quadratic function f(x) attains a maximum of 3 at x = 1. The value of the function at
x = 0 is 1. What is the value of f(x) at x = 10?
(1) –119
(2) –159
(3) –110
(4) –180
(5) –105
Solution:
Let f(x) = px2 + qx + k, where p, q and k are integers
f(0) = k = 1
f(x) = px2 + qx + 1
f(x) = px2 + qx + k
f’(x) = 2px + q
When f’(x) = 0, x q/2p
f(x) attains maximum at x = 1
q 2p
f(1) = p + q + 1 = 3
1 – p = 3
p 2
q = 4
f(x) 2×2 + 4x + 1
f(10) = 200 40 1 159
Hence, option 2.
3.Two circles with centres P and Q cut each other at two distinct points A and B. The
circles have the same radii and neither P nor Q falls within the intersection of the
circles. What is the smallest range that includes all possible values of the angle AQP
in degrees?
(1) Between 0 and 90
(2) Between 0 and 30
(3) Between 0 and 60
(4) Between 0 and 75
(5) Between 0 and 45
Solution:
P and Q do not lie within the intersection of the two circles.
So they lie on the circumferences or outside the circumferences. If they lie on the
circumferences then APQ forms an equilateral triangle
So, m AQP = 60o
From the diagram, if they lie outside the circumferences, m AQ’P’ 60o
Also, m AQP would be 0o if A, Q and P were collinear.
But as P and Q cut each other in two distinct points, A, Q and P cannot be collinear.
m AQP > 0o
m AQP lies between 0o and 60o
Hence, option 3.
Directions for Questions 4 and 5:
Let S be the set of all pairs (i, j) where 1 = i < j = n and n = 4. Any two distinct members of S are called “friends” if they have one constituent of the pairs in common and “enemies” otherwise For example if n = 4, then S = {(1, 2), (1, 3), (1, 4), (2, 3), (2, 4), (3, 4)}. Here, (1, 2) a nd (1, 3) are friends, (1, 2) and (2, 3) are also friends, but (1, 4) and (2, 3) are enemies. 4.For general n, how many enemies will each member of S have?
(1) n – 3
(2)
(3) 2n – 7
(4)
(5)
Solution:
Enemies of every pair are the pairs formed with all numbers other than the two in
the member itself.
If there are n elements then each member has
Hence, option 4.
5.For general n, consider any two members of S that are friends. How many other
members of S will be common friends of both these members?
(1)
(2) 2n – 6
(3)
(4) n – 2
(5)
Solution:
Two members are friends if they have one element in common.
All the members having one constituent as the common element are common
friends.
There are (n – 3) such friends.
Also, one pair formed by the uncommon constituents of the two friends is a common
friend.
There are n – 3 + 1 = n – 2 common friends.
Hence, option 4.
Directions for Questions 6 and 7:
Shabnam is considering three alternatives to invest her surplus cash for a week. She wishes
to guarantee maximum returns on her investment. She has three options, each of which can
be utilized fully or partially in conjunction with others.
Option A: Invest in a public sector bank. It promises a return of +0.10%
Option B: Invest in mutual funds of ABC Ltd. A rise in the stock market will result in a
return of +5%, while a fall will entail a return of –3%
Option C: Invest in mutual funds of CBA Ltd. A rise in the stock market will result in a
return of –2.5%, while a fall will entail a return of +2%
6.The maximum guaranteed return to Shabnam is:
(1) 0.25%
(2) 0.10%
(3) 0.20%
(4) 0.15%
(5) 0.30%
Solution:
Let Shabnam have Rs. 100 to invest. Let Rs. x, Rs. y and Rs. z be invested in option A,
B and C respectively.
x + y + z = 100 … (I)
If there is a rise in the stock market, returns = 0.001x + 0.05y – 0.025z
If there is a fall in the stock market, returns = 0.001x – 0.03y + 0.02z
Now, x, y and z should be such that regardless of whether the market rises or falls,
they give the same return, which is the maximum guaranteed return.
0.001x + 0.05y – 0.025z = 0.001x – 0.03y + 0.02z
y/z = 9/16
Now, consider different possible values of x, y and z. The returns are as follows:
x y z Returns = 0.001x + 0.05y
– 0.025z
75 9 16 0.125
50 18 32 0.15
25 27 48 0.175
0 36 64 0.2
We see that as the values of y and z increase, the returns increase.
The returns are maximum when x = 0%, y = 36% and z = 64%
(Note that the values of y and x are multiples of 9 and 16.)
The maximum returns are 0.2%.
Hence, option 3.
7. What strategy will maximize the guaranteed return to Shabnam?
(1) 100% in option A
(2) 36% in option B and 64% in option C
(3) 64% in option B and 36% in option C
(4) 1/3 in each of the three options
(5) 30% in option A, 32% in option B and 38% in option C
Solution:
As shown by the table formulated in the first question, maximum returns are
guaranteed by investing 36% in option B and 64% in option C.
Hence, option 2.
Directions for Questions 8 and 9:
Cities A and B are in different time zones. A is located 3000 km east of B. The table below
describes the schedule of an airline operating non-stop flights between A and B. All the
times indicated are local and on the same day.
Departure Arrival
City Time City Time
B 8:00 a.m. A 3:00 p.m.
A 4:00 p.m. B 8: p.m.
Assume that planes cruise at the same speed in both directions. However, the
effective speed is influenced by a steady wind blowing from east to west at 50 km
per hour.
8.What is the time difference between A and B?
(1) 1 hour and 30 minutes
(2) 2 hours
(3) 2 hours and 30 minutes
(4) 1 hour
(5) Cannot be determined
Solution:
Let the speed of the plane be x kmph.
Then the speed from B to A is (x – 50) kmph and that from A to B is (x + 50) kmph.
Note that the plane travels from B to A, halts for 1 hour and travels back to B, all in
12 hrs.
3000/(x – 50) + 1 + 3000/(x + 50) = 12
Now consider options for this question. We can easily see that x = 550 satisfies the
above expression.
Speed of plane = 550 kmph
Now, the plane takes 3000/500 = 6 hrs to travel from B to A.
It reaches A when the time at B is 8:00 am + 6 hrs = 2:00 p.m.
=> The time difference between A and B is 1 hour.
Hence, option 4.
9.What is the plane’s cruising speed in km per hour?
(1) 700
(2) 550
(3) 600
(4) 500
(5) Cannot be determined
Solution:
As calculated in the first question, the cruising speed of the plane is 550 kmph.
Hence, option 2.
10.Consider all four digit numbers for which the first two digits are equal and the last
two digits are also equal. How many such numbers are perfect squares?
(1) 3
(2) 2
(3) 4
(4) 0
(5) 1
Solution:
Let aabb (a 0 a and b being single digits) be a perfect square.
aabb = 1100a + 11b = 11(100a + b)
Also, as aabb is a perfect square, it is a multiple of 121.
aabb = 121K, where K is also perfect square.
For K = 4, aabb is a 3 digit number, while for K > 81, K is a 5 digit number.
For 81 = K = 9
121 × 9 = 1089
121 × 16 = 1936
121 × 25 = 3025
121 × 36 = 4356
121 × 49 = 5929
121 × 64 = 7744
121 × 81 = 9801
There is only one number 7744 of the form aabb, which is a perfect square.
Hence, option 5.
11.In a tournament, there are n teams T1 , T2 ….., Tn with n > 5. Each team consists of k
players, k > 3. The following pairs of teams have one player in common:
T1 & T2 , T2 & T3 ,……, Tn 1 & Tn , and Tn & T1.
No other pair of teams has any player in common. How many players are
participating in the tournament, considering all the n teams together?
(1) n(k – 1)
(2) k(n – 1)
(3) n(k – 2)
(4) k(k – 2)
(5) (n – 1)(k – 1)
Solution:
Each team has (k – 2) players to itself and shares 2 players with other two teams.
n pairs of teams have 1 player in common and there are n teams.
Total number of players = n(k – 2) + n
= nk – 2n + n
= nk – n
= n(k – 1)
Hence, option 1.
Directions for Questions 12 and 13:
Mr. David manufactures and sells a single product at a fixed price in a niche market. The
selling price of each unit is Rs. 30. On the other hand, the cost, in rupees, of producing x
units is 240 + bx + cx2, where b and c are some constants. Mr. David noticed that doubling
the daily production from 20 to 40 units increases the daily production cost by 66.66%.
However, an increase in daily production from 40 to 60 units results in an increase of only
50% in the daily production cost. Assume that demand is unlimited and that Mr. David can
sell as much as he can produce. His objective is to maximize the profit.
12.How many units should Mr. David produce daily?
(1) 130
(2) 100
(3) 70
(4) 150
(5) Cannot be determined
Solution:
The cost function C(x) = 240 + bx + cx2
C(20) = 240 + 20b + 400c
C(40) = 240 + 40b + 1600c
C(60) = 240 + 60b + 3600c
By conditions,
2/3 × C(20) = C(40) – C(20)
C(40) = 5/3 × C(20)
240 + 40b + 1600c = 400 + 100b/3 + 2000c/3
20b/3 + 2800c/3 = 160
20b + 2800c = 480 … (I)
Also,
½ × C(40) = C(60) – C(40)
3/2 × C(40) = C(60)
360 + 60b + 2400c = 240 + 60b + 3600c
c =1/10
b = 10 … (from I)
Profit for x units is 30x – C(x)
P(x) = 30x – 240 – 10x – x2/10 = –240 + 20x – x2/10
The derivative of P(x) = P’(x) = 20 – x/5
For maximization of profit P’(x) = 0 and P’’(x) < 0 When P’(x) is zero, 20 – x/5 = 0 x = 100 P’’(x) = –0.2 Hence, option 2. 13.What is the maximum daily profit, in rupees, that Mr. David can realize from his
business?
(1) 620
(2) 920
(3) 840
(4) 760
(5) Cannot be determined
Solution:
Following from the first question, at x = 100 the profit is maximum.
At that level of production
P(x) = –240 + 20 (100) – (100)2/10
= –240 + 2000 – 1000
= 760
Hence, option 4.
Directions for Questions 14 and 15:
Let a1 = p and b1 = q, where p and q are positive quantities.
Define:
an = pbn – 1 ; bn = qbn 1 (for even n > 1)
and an = pan – 1 ; bn = qan 1 (for odd n > 1)
14.
(1)
(2)
(3)
(4)
(5)
Solution:
We have the following for different values of n
n an bn
1 p q
2 pq q2
3 p2q pq2
4 p2q2 pq3
5 p3q2 p2q3
6 p3q3 p2q4
7 p4q3 p3q4
For even n (say, n = 4),
Now consider the given options, for n = 4
(1) gives pq²(p + q)²
(2) gives pq²(p + q)
(3) gives pq(p + q)
(4) gives q²(p + q)
(5) gives q²(p + q)²
Hence, option 2.
15.If p = 1/3 and q = 2/3, then what is the smallest odd n such that an + bn < 0.01? (1) 7 (2) 13 (3) 11 (4) 9 (5) 15 Solution:
For odd n, an + bn = p(n + 1)/2 q(n – 1)/2 + q(n + 1)/2 p(n – 1)/2
= p(n – 1)/2 q(n – 1)/2 (p + q)
Here, p = 1/3, q = 2/3
p + q = 1
an + bn = p(n – 1)/2 q(n – 1)/2 = (2/9)(n – 1)/2
Now considering options starting from the lowest,
For n = 7, an + bn = 8/729 = 1/91 > 1/100
For n = 9, an + bn = 16/6561 = 1/410 < 1/100 Hence, option 4. Directions for Questions 16 through 19: Each question is followed by two statements A and B. Answer each question using the following instructions. Mark (1) if the question can be answered by using statement A alone but not by using statement B alone. Mark (2) if the question can be answered by using statement B alone but not by using statement A alone. Mark (3) if the question can be answered by using both the statements together but not by using either of the statements alone. Mark (4) if the question cannot be answered on the basis of the two statements. 16.The average weight of a class of 100 students is 45 kg. The class consists of two
sections, I and II, each with 50 students. The average weight, WI, of Section I is
smaller than the average weight, WII, of Section II. If the heaviest student, say
Deepak, of Section II is moved to Section I, and the lightest student, say Poonam, of
Section I is moved to Section II, then the average weights of the two sections are
switched, i.e., the average weight of Section I becomes WII and that of Section II
becomes WI. What is the weight of Poonam?
A. WII – WI = 1.0
B. Moving Deepak from Section II to I (without any move from I to II) makes the
average weights of the two sections equal.
Solution:
Let the weights of Deepak and Poonam be d and p respectively.
(50WII + 50WI)/100 = 45
WII + WI = 90 ______ (i)
50WI + d – p = 50WII
50(WII – WI) = d – p ______ (ii)
From Statement A, WII – WI = 1 ______ (iii)
From (i), (ii) and (iii),
WI and WII can be found.
Also, d – p = 50 ______ (iv)
However this information does not give us the value of p. Statement A is insufficient
to answer the question.
From Statement B,
49(WI + d) = 51(WII – d)
51WII – 49WI = 100d ______ (v)
This alone cannot help us find the value of p. Statement B is insufficient to answer
the question.
Considering both statements together, we have values of WI and WII, which can be
substituted in (v) to find d, which can be used to find p using (iv).
Hence, option 3.
17.Consider integers x, y and z. What is the minimum possible value of x2 + y2 + z2 ?
A. x + y + z = 89
B. Among x, y, z two are equal.
Solution:
Statement A:
x + y + z = 89
x2 + y2 + z2 will be minimum when x = y = z = 89/3
But 89/3 is a non-integer.
We consider integer values of x, y, z which are as close as possible to 89/3.
We get two cases:
1. x, y, z = 30, 30, 29
x2 + y2 + z2 = 2641
2. x, y, z = 31, 29, 29
x2 + y2 + z2 = 2643
Minimum possible value of x2 + y2 + z2 is 2641. Thus statement A is sufficient to get
the answer. Though statement B states a fact related to the minimum value, it is not
necessary to arrive at the minimum value.
Hence, option 1.
18. Rahim plans to draw a square JKLM with a point O on the side JK but is not
successful. Why is Rahim unable to draw the square?
A. The length of OM is twice that of OL.
B. The length of OM is 4 cm.
Solution:
Let p be the side of square JKLM.
From Statement A,
OM = 2 × OL
OM is maximum when it is the diagonal of the square and has length ( 2)p
When OM is maximum, OM = ( 2)OL
OM 2 × OL if O lies on JK
Rahim is unable to draw the square.
Hence, option 1.
19. ABC Corporation is required to maintain at least 400 Kilolitres of water at all times
in its factory, in order to meet safety and regulatory requirements. ABC is
considering the suitability of a spherical tank with uniform wall thickness for the
purpose. The outer diameter of the tank is 10 meters. Is the tank capacity adequate
to meet ABC’s requirements?
A. The inner diameter of the tank is at least 8 meters.
B. The tank weighs 30,000 kg when empty, and is made of a material with density
of 3 gm/cc.
Solution:
Let the inner radius be r meter. Capacity of tank = (1 m3 = 1 kilolitre)
From statement A, since r = 4m
Capacity of tank > 256 m3
Since the capacity needed is more than 256 m 3 statement A is insufficient.
From statement B,
Volume of the material of tank = mass/density = 30000kg/(3 gm/cc) = 10,000,000
cm3 = 10 m3
Hence the inner volume of tank = Outer volume – Volume of material of tank
Therefore, we can say that the tank capacity is adequate.
Hence, option 2.
20. Suppose you have a currency, named Miso, in three denominations: 1 Miso, 10 Misos
and 50 Misos. In how many ways can you pay a bill of 107 Misos?
(1) 17
(2) 16
(3) 18
(4) 15
(5) 19
Solution:
The bill can be paid in 18 ways as shown in the given table. Hence, option 3.
50 Misos 10 Misos 1 Miso Total
0 0
107 107
0 1
97 107
0 2
87 107
0 3
77 107
0 4
67 107
0 5
57 107
0 6
47 107
0 7
37 107
0 8
27 107
0 9
17 107
0 10
7 107
1 0
57 107
1 1
47 107
1 2
37 107
1 3
27 107
1 4
17 107
1 5
7 107
2 0
7 107
21. How many pairs of positive integers m, n satisfy 1/m + 4/n = 1/12 where n is an
odd integer less than 60?
(1) 6
(2) 4
(3) 7
(4) 5
(5) 3
Solution:
1/m + 4/n = 1/12
1/m = 1/12 – 4/n
m = 12n / (n – 48)
As, m is a positive integer, n should be greater than 48 and moreover since n is a
positive odd integer lesser than 60, n can take values 49, 51, 53, 55, 57 and 59.
If n = 49, 51, 57 then m is a positive integer.
If n = 53, 55, 59 then m is not an integer.
3 pairs of values of m and n satisfy the given equation.
Hence, option 5.
22.A confused bank teller transposed the rupees and paise when he cashed a cheque
for Shailaja, giving her rupees instead of paise and paise instead of rupees. After
buying a toffee for 50 paise, Shailaja noticed that she was left with exactly three
times as much as the amount on the cheque. Which of the following is a valid
statement about the cheque amount?
(1) Over Rupees 13 but less than Rupees 14
(2) Over Rupees 7 but less than Rupees 8
(3) Over Rupees 22 but less than Rupees 23
(4) Over Rupees 18 but less than Rupees 19
(5) Over Rupees 4 but less than Rupees 5
Solution:
Let the amount on Shailaja’s cheque be Rs x and paise y = (100x + y) paise (x and y
are positive integers)
The teller gives her (100y + x) paise.
Now, 100y + x – 50 = 3(100x + y)
97y – 299x = 50
y = (50 + 299x)/97
= [50 + 8x + 291x]/97
= [(50 + 8x)/97] + 3
Now as y is an integer, (50 + 8x) has to be a multiple of 97 with x, y = 99
50 + 8x = 97k (k is an integer)
x = 12k – 6 + [(k – 2)/8]
k 2 10 18
x 18 115 212
x = 18 is the only possible value.
This implies that y = 5
The amount on Shailaja’s cheque is over Rs 18 but less than Rs 19
Hence, option 4.
23.Consider the set S = {2, 3, 4, …., 2n + 1}, where n is a positive integer larger than
2007. Define X as the average of the odd integers in S and Y as the average of the
even integers in S. What is the value of X – Y?
(1) 0
(2) 1
(3) n/2
(4) n + 1/2n
(5) 2008
Solution:
Y (2 4 6 8 2n)/n
X (3 5 7 9 (2 n + 1))/n
((2 1) (4 1) (6 1) (8 1) (2n + 1))/n
(2 4 6 8 2n)/n (1 1 1 1 n times)/n
= Y + 1
X – Y = 1
Hence, option 2.
Note: The information that ‘n is a positive integer larger than 2007′ does not affect
the answer in any way.
24.Ten years ago, the ages of the members of a joint family of eight people added up to
231 years. Three years later, one member died at the age of 60 years and a child was
born during the same year. After another three years, one more member died, again
at 60, and a child was born during the same year. The current average age of this
eight member joint family is nearest to:
(1) 23 years
(2) 22 years
(3) 21 years
(4) 25 years
(5) 24 years
Solution:
The sum of the ages of the members of the family ten years ago = 231
The sum of the ages of the members of the family seven years ago = 231 + (3 × icon cool CAT 2007 Question Paper with Solutions –
60 = 195
The sum of the ages of the members of the family four years ago = 195 + (3 × icon cool CAT 2007 Question Paper with Solutions –
60 = 159
The sum of the ages of the members of the family now = 159 + (4 × icon cool CAT 2007 Question Paper with Solutions = 191
Required average = 191/8 = 23.875 24
Hence, option 5.
25. A function f(x) satisfies f (1) = 3600, and f(1) + f(2) + … + f(n) = n² f(n), for all
positive integers n >1. What is the value of f(9) ?
(1) 80
(2) 240
(3) 200
(4) 100
(5) 120
Solution:
f(1) + f(2) + f (3) f(n 1) + f(n) = n2f(n) … (i)
Similarly, f(1) + f(2) + f (3) f(n 1) ( n 1)2 f(n 1) … (ii)
Subtracting equation (ii) from (i),
f(n) = n2 f(n) – (n – 1)2f(n 1)
(n2 – 1)f(n) = (n – 1)2f(n – 1)
Hence, option 1.
SECTION 2- This section contains 25 questions

Directions for Questions 26 to 29: Each question is followed by two statements, A and B.
Answer each question using the following instructions:
Mark (1) if the question can be answered by using the statement A alone but not by using
the statement B alone.
Mark (2) if the question can be answered by using the statement B alone but not by using
the statement A alone.
Mark (3) if the question can be answered by using either of the statements alone.
Mark (4) if the question can be answered by using both the statements together but not by
either of the statements alone.
Mark (5) if the question cannot be answered on the basis of the two statements.
26. In a football match, at half-time, Mahindra and Mahindra Club was trailing by three
goals. Did it win the match?
A. In the second half Mahindra and Mahindra Club scored four goals.
B. The opponent scored four goals in the match.
Solution:
From Statement A, the MM club scored 4 goals in the second half. The number of
goals scored by the opponent is not known. So the winner cannot be determined.
Statement A is insufficient.
From Statement B, the opponent scored 4 goals in the match, but we do not know
the number of goals that the MM club scored. Statement B is insufficient.
Considering both the statements we have the following.
First Half Second Half Final Score
MM Club Opponent MM Club Opponent MM Club Opponent
0 3 4 1 4 4
1 4 4 0 5 4
Thus, MM club could have won the match or could have tied it. The question cannot
be answered.
Hence, option 5.
27. In a particular school, sixty students were athletes. Ten among them were also
among the top academic performers. How many top academic performers were in
the school?
A. Sixty per cent of the top academic performers were not athletes.
B. All the top academic performers were not necessarily athletes.
Solution:
From Statement A, 40% of the top academic performers were athletes.
If there are x top academic performers, 10 = 0.4x
x = 25
Statement A is sufficient.
Statement B does not give any useful information.
Hence, option 1.
28. Five students Atul, Bala, Chetan, Dev and Ernesto were the only ones who
participated in a quiz contest. They were ranked based on their scores in the
contest. Dev got a higher rank as compared to Ernesto, while Bala got a higher rank
as compared to Chetan Chetan’s rank was lower than the median Who among the
five got the highest rank?
A. Atul was the last rank holder.
B. Bala was not among the top two rank holders.
Solution:
Chetan’s rank 4 or 5
Now, Bala < Chetan and Dev < Ernesto. From Statement A, Case 1 Case 2 Case 3 1 Bala Dev Dev 2 Dev Ernesto Bala 3 Ernesto Bala Ernesto 4 Chetan Chetan Chetan 5 Atul Atul Atul The highest rank holder cannot be determined. Statement A is insufficient. Statement B is also insufficient. Considering both statements together (refer to the table), Case 2 holds. Dev got the highest rank. Hence, option 4. 29. Thirty per cent of the employees of a call centre are males. Ten per cent of the
female employees have an engineering background. What is the percentage of male
employees with engineering background?
A. Twenty five per cent of the employees have engineering background.
B. Number of male employees having an engineering background is 20% more than
the number of female employees having an engineering background.
Solution:
Let there be 100x employees.
So, 30x are male and 70x are female.
7x female employees have an engineering background.
From statement A, 25x employees have an engineering background.
18x male employees have an engineering background.
Required percentage = 18x × 100/ 30x
Statement A is sufficient.
From Statement B, Number of male employees having an engineering background
= 1.2 × 7x
Required percentage = 1.2 × 7x × 100/30x
Statement B is also sufficient.
Hence, option 3.
DIRECTIONS for Questions 30 to 33: Answer the following questions based on the information given below:
The proportion of male students and the proportion of vegetarian students in a school are
given below. The school has a total of 800 students, 80% of whom are in the Secondary
Section and rest equally divided between Class 11 and 12:
Male (M) Vegetarian (V)
Class 12
0.6
Class 11
0.55
0.5
Secondary Section
0.55
Total
0.475
0.53
30. What is the percentage of male students in the secondary section?
(1) 40
(2) 45
(3) 50
(4) 55
(5) 60
Solution:
From the table given in the question,
Total students = 800
Students in Secondary
= 0.8 × 800
= 640
Students in Class 11
= (800 – 640)/2
= 80
Students in Class 12 = 80
Males in Class 11
= 0.55 × 80
= 44
Males in Class 12
= 0.6 × 80
= 48
Males in Secondary
= 0.475 × 800 – 44 – 48
= 288
Vegetarians in Class 11
= 0.5 × 80
= 40
Vegetarians in Secondary
= 0.55 × 640
= 352
Vegetarians in Class 12
= 800 × 0.53 – 40 – 352
= 32
Now, the percentage of male students in secondary section
= 288 × 100/640
= 45%
Hence, option 2.
31. In Class 12, twenty five per cent of the vegetarians are male. What is the difference
between the number of female vegetarians and male non-vegetarians?
(1) less than 8
(2) 10
(3) 12
(4) 14
(5) 16
Solution:
Vegetarian Males in Class 12
= 0.25 × 48
= 12
Non-vegetarian Males in class 12 = 36
Vegetarian females in class 12
= Vegetarians in class 12 – Male vegetarians in class 12
= 32 – 12 = 20
(We derived the number of vegetarians in the class in the previous question)
Required difference = 36 – 20 = 16
Hence, option 5.
32. What is the percentage of vegetarian students in Class 12?
(1) 40
(2) 45
(3) 50
(4) 55
(5) 60
Solution:
In the first question we derived the number of vegetarians in class 12 as 32 and the
total number of students in the class as 80.
The percentage of vegetarians in class 12 = 32 × 100/80 = 40%
Hence, option 1.
33. In the Secondary Section, 50% of the students are vegetarian males. Which of the
following statements is correct?
(1) Except vegetarian males, all other groups have same number of students.
(2) Except non-vegetarian males, all other groups have same number of students.
(3) Except vegetarian females, all other groups have same number of students.
(4) Except non-vegetarian females, all other groups have same number of students
(5) All of the above groups have the same number of students.
Solution:
(This question was not considered for evaluation as there was an error in the
question)
“50% of the students are vegetarian males” contradicts the data given initially
Interpreting it as “50% of the males are vegetarian” we have the following :
In secondary,
Vegetarian males = 144
Non-vegetarian males = 144
Vegetarian females = 352 –144 = 208
Non-vegetarian females = 352 – 208 = 144
Except vegetarian females, all other groups have same number of students.
Hence, option 3.
Directions for Questions 34 to 37: Answer the following questions based on the
information given below.

The following table shows the break-up of actual costs incurred by a company in last five
years (year 2002 to year 2006) to produce a particular product.
The production capacity of the company is 2000 units. The selling price for the year 2006
was Rs. 125 per unit. Some costs change almost in direct proportion to the change in
volume of production, while others do not follow any obvious pattern of change with
respect to the volume of production and hence are considered fixed. Using the information
provided for the year 2006 as the basis for projecting the figures for the year 2007, answer
the following questions.
Year
Year
Year
Year
Year
2002
2003
2004
2005
2006
Volume of production
1000 900 1100 1200 1200
Costs (Rs.)
Material
50,000 45,100 55,200 59,900 60,000
Labour
20,000 18,000 22,100 24,150 24,000
Consumables
2,000 2,200 1,800 1,600 1,400
Rent of building
1,000 1,000 1,100 1,100 1,200
Rates and taxes
400 400 400 400 400
Repair and maintenance expenses 800 820 780 790 800
Operating cost of machines 30,000 27,000 33,500 36,020 36,000
Selling and marketing expenses 5,750 5,800 5,800 5,750 5,800
34. What is the approximate cost per unit in rupees, if the company produces and sells
1400 units in the year 2007?
(1) 104
(2) 107
(3) 110
(4) 115
(5) 116
Solution:
Observing the values through the years, we can say that Material, Labour and
Operating costs directly vary with the change in volume of production.
The other costs are almost constant.
If the production is x units, the variable cost for material, labour and operation is
50x, 20x and 30x respectively.
Total variable cost = 100x
Total fixed cost (using information for 2006)
= 1400 + 1200 + 400 + 800 + 5800
= 9600
Total cost of producing x units = 100x + 9600
Now, x = 1400
Cost per unit = (1400 × 100 + 9600)/1400
= 106.85
Hence, option 2.
35. What is the minimum number of units that the company needs to produce and sell
to avoid any loss?
(1) 313
(2) 350
(3) 384
(4) 747
(5) 928
Solution:
From the explanation given in the first question, to avoid any loss, 100x 9600 =
125x
x = 384
Hence, option 3.
36. If the company reduces the price by 5%, it can produce and sell as many units as it
desires. How many units should the company produce to maximize its profit?
(1) 1400
(2) 1600
(3) 1800
(4) 1900
(5) 2000
Solution:
The new reduced price
= 0.95 × 125
= 118.75
Profit
= 118.75x – 100x – 9600
= 18.75x – 9600
Profit will be maximum when 18.75x is maximum. As the maximum production
capacity is 2000 units, profit is maximum when 2000 units are produced.
Hence, option 5.
37. Given that the company cannot sell more than 1700 units, and it will have to reduce
the price by Rs. 5 for all units, if it wants to sell more than 1400 units, what is the
maximum profit, in rupees, that the company can earn?
(1) 25,400
(2) 24,400
(3) 31,400
(4) 32,900
(5) 32,000
Solution:
Profit for 1400 units
= 1400 × 125 – (1400 × 100 + 9600)
= 25400
Profit for (1400 + m) units
= (1400 + m) × 120 – ((1400 + m) × 100 + 9600)
= 18400 + 20m
Maximum value of m = 300
Maximum profit for 1400 + 300 units = 24400
Maximum profit that the company can earn is 25400.
Hence, option 1.
Directions for Questions 38 to 41: Answer the following questions based on the
information given below.
The table below shows the comparative costs, in US Dollars, of major surgeries in USA and
a select few Asian countries.
Procedure Comparative Costs in USA and some Asian Countries
(in US Dollars)
USA India Thailand Singapore Malaysia
Heart Bypass 130000 10000 11000 18500 9000
Heart Valve Replacement 160000 9000 10000 12500 9000
Angioplasty 57000 11000 13000 13000 11000
Hip Replacement 43000 9000 12000 12000 10000
Hysterectomy 20000 3000 4500 6000 3000
Knee Replacement 40000 8500 10000 13000 8000
Spinal Fusion 62000 5500 7000 9000 6000
The equivalent of one US Dollar in the local currencies is given below.
1 US Dollar Equivalent
India 40.928 Rupees
Malaysia 3.51 Ringits
Thailand 32.89 Bahts
Singapore 1.53 $ Dollars
A consulting firm found that the quality of the health services were not the same in all the
countries above. A poor quality of a surgery may have significant repercussions in future,
resulting in more cost in correcting mistakes. The cost of poor quality of surgery is given in
the table below.
Procedure Comparative Costs in USA and some Asian Countries
(in US Dollars ‘000)
USA India Thailand Singapore Malaysia
Heart Bypass 0 3 3 2 4
Heart Valve Replacement 0 5 4 5 5
Angioplasty
0 5 5 4 6
Hip Replacement 0 7 5 5 8
Hysterectomy 0 5 6 5 4
Knee Replacement 0 9 6 4 4
Spinal Fusion 0 5 6 5 6
38. A US citizen is hurt in an accident and requires an angioplasty, hip replacement and
a knee replacement. Cost of foreign travel and stay is not a consideration since the
government will take care of it. Which country will result in the cheapest package,
taking cost of poor quality into account?
(1) India
(2) Thailand
(3) Malaysia
(4) Singapore
(5) USA
Solution:
As shown in the table, Malaysia will have the cheapest package.
Hence, option 3.
India Thailand Singapore Malaysia USA
Angioplasty 16000 18000 17000 17000 57000
Hip Replacement 16000 17000 17000 18000 43000
Knee Replacement 17500 16000 17000 12000 40000
Total
49500 51000 51000 47000 140000
39.Taking the cost of poor quality into account, which country/countries will be the
most expensive for knee replacement?
(1) India
(2) Thailand
(3) Malaysia
(4) Singapore
(5) India and Singapore
Solution:
Referring to the table formulated in the first question, India will be the most
expensive for knee replacement.
Hence, option 1.
40. Approximately, what difference in amount in Bahts will it make to a Thai citizen if
she were to get a hysterectomy done in India instead of in her native country, taking
into account the cost of poor quality? It costs 7500 Bahts for one-way travel
between Thailand and India.
(1) 23500
(2) 40500
(3) 57500
(4) 67500
(5) 75000
Solution:
Cost of Hysterectomy in Thailand
= 4500 + 6000
= 10500 USD
Cost of Hysterectomy in India
= 3000 + 5000
= 8000 USD
Travelling cost
= 15000 Bahts
= 15000/32.89 USD
= 456 USD
Required difference
= 10500 – 8456
= 2044 USD
= 2044 × 32.89
= 67227 Bahts
Hence, option 4.
41. The rupee value increases to Rs. 35 for a US Dollar, and all other things, including
quality, remain the same. What is the approximate difference in cost, in US Dollars,
between Singapore and India for a Spinal Fusion, taking this change into account?
(1) 700
(2) 2500
(3) 4500
(4) 8000
(5) No difference
Solution:
Cost of spinal fusion in India = Rs. 5500 × 40.928
Cost with the increased value of Rupee = 5500 × 40.928/35 = 6431 USD
Cost of Spinal Fusion in Singapore = 9000 USD
Required difference = 9000 – 6431 = 2569 USD
Hence, option 2.
Directions for Questions 42 to 46: Answer the following questions based on the
information given below.
A low-cost airline company connects ten Indian cities, A to J. The table below gives the
distance between a pair of airports and the corresponding price charged by the company.
Travel is permitted only from a departure airport to an arrival airport. The customers do
not travel by a route where they have to stop at more than two intermediate airports.
Airport of
Distance between the
Sector No. Airport of
Departure
Arrival
Airports (km.) Price (Rs.)
1 A B 560 670
2 A C 790 1350
3 A D 850 1250
4 A E 1245 1600
5 A F 1345 1700
6 A G 1350 2450
7 A H 1950 1850
8 B C 1650 2000
9 B H 1750 1900
10 B I 2100 2450
11 B J 2300 2275
12 C D 460 450
13 C F 410 430
14 C G 910 1100
15 D E 540 590
16 D F 625 700
17 D G 640 750
18 D H 950 1250
19 D J 1650 2450
20 E F 1250 1700
21 E G 970 1150
22 E H 850 875
23 F G 900 1050
24 F I 875 950
25 F J 970 1150
26 G I 510 550
27 G J 830 890
28 H I 790 970
29 H J 400 425
30 I J 460 540
42. What is the lowest price, in rupees, a passenger has to pay for travelling by the
shortest route from A to J?
(1) 2275
(2) 2850
(3) 2890
(4) 2930
(5) 3340
Solution:
Possible routes from A to J are as shown in the table below.
Route Distance Price
ABJ 2860 2945
ADJ 2500 3700
AFJ 2315 2850
AGJ 2180 3340
AHJ 2350 2275
ABHJ 2710 2995
ABIJ 3120 3660
ACDJ 2900 4250
ACFJ 2170 2930
ACGJ 2530 3340
ADFJ 2445 3100
ADGJ 2320 2890
ADHJ 2200 2925
AEFJ 3465 4450
AEGJ 3045 3640
AEHJ 2495 2900
AFGJ 3075 3640
AFIJ 2680 3190
AGIJ 2320 3540
AHIJ 3200 3360
The shortest distance is by the route A-C-F-J.
The price is 1350 + 430 + 1150 = Rs. 2930
Hence, option 4.
43. The company plans to introduce a direct flight between A and J. The market
research results indicate that all its existing passengers travelling between A and J
will use this direct flight if it is priced 5% below the minimum price that they pay at
present. What should the company charge approximately, in rupees, for this direct
flight?
(1) 1991
(2) 2161
(3) 2707
(4) 2745
(5) 2783
Solution:
The current market price paid by the customers is Rs. 2275 (A-H-J).
Therefore, the company should charge (2275 × 0.95) = Rs. 2161.25
Hence, option 2.
44. If the airports C, D and H are closed down owing to security reasons, what would be
the minimum price, in rupees, to be paid by a passenger travelling from A to J?
(1) 2275
(2) 2615
(3) 2850
(4) 2945
(5) 3190
Solution:
If C, D and H are closed, the cheapest route will be A-F-J and it will cost Rs. 2850.
Hence, option 3.
45.If the prices include a margin of 10% over the total cost that the company incurs,
what is the minimum cost per kilometer that the company incurs in flying from A to
J?
(1) 0.77
(2) 0.88
(3) 0.99
(4) 1.06
(5) 1.08
Solution:
The minimum cost per km that the company incurs would correspond to the
minimum price per km route.
By observation from the table, minimum price per kilometre is for the route AHJ
and is equal to 2275/2350 = 0.97
Minimum cost per kilometre = 0.97/1.1 = 0.88
Hence, option 2.
46.If the prices include a margin of 15% over the total cost that the company incurs,
which among the following is the distance to be covered in flying from A to J that
minimizes the total cost per kilometer for the company?
(1) 2170
(2) 2180
(3) 2315
(4) 2350
(5) 2390
Solution:
Even if the margin for the prices changes the minimum cost per km would
correspond to the same route namely A-H-J.
From the table, the distance for the travel = 2350 km
Hence, option 4.
Directions for Questions 47 to 50: Answer the following questions based on the
information given below.
A health-drink company’s R&D department is trying to make various diet formulations,
which can be used for certain specific purposes. It is considering a choice of 5 alternative
ingredients (O, P, Q, R, and S), which can be used in different proportions in the
formulations. The table below gives the composition of these ingredients. The cost per unit
of each of these ingredients is O: 150, P: 50, Q: 200, R: 500, S: 100.
Composition
Ingredient Carbohydrate % Protein % Fat % Minerals %
O 50 30 10 10
P 80 20 0 0
Q 10 30 50 10
R 5 50 40 5
S 45 50 0 5
47. For a recuperating patient, the doctor recommended a diet containing 10% minerals
and at least 30% protein. In how many different ways can we prepare this diet by
mixing at least two ingredients?
(1) One
(2) Two
(3) Three
(4) Four
(5) None
Solution:
The diet should contain 10% minerals. P contains no minerals.
P cannot be a part of any mixture.
R and S both contain 5% minerals.
Mix of R and S in any proportion cannot give 10% minerals.
Consider O and R in the proportion x:y
10x + 5y = 10(x + y)
5y = 10y, which is not possible.
Similarly, Q and S, O and S, and Q and R are not possible.
Similarly a mix of three ingredients is not possible.
The only possible mix is that of O and Q in equal proportion.
Hence, option 1.
48. Which among the following is the formulation having the lowest cost per unit for a
diet having 10% fat and at least 30% protein? The diet has to be formed by mixing
two ingredients.
(1) P and Q
(2) P and S
(3) P and R
(4) Q and S
(5) R and S
Solution:
Consider the options.
Option 1:
P and Q have to be mixed in the proportion 4:1 to achieve 10% fat content. But this
does not give 30% protein.
Option 2:
P and S do not contain fat.
Option 3:
P and R should be mixed in the proportion 3:1 to achieve 10% fat content. But 30%
protein content is not achieved.
Option 4:
Q and S should be mixed in the proportion 1:4 to achieve 10% fat content and 46%
protein content. The cost of this mix per unit would be 6/5.
Option 5:
R and S should be mixed in the proportion 1:3 to achieve 10% fat content and 50%
protein content. The cost per unit of this mix would be 2.
Therefore, lowest cost is for Q and S.
Hence, option 4.
49. In what proportion should P, Q and S be mixed to make a diet having at least 60%
carbohydrate at the lowest per unit cost?
(1) 2 : 1 : 3
(2) 4 : 1 : 2
(3) 2 : 1 : 4
(4) 3 : 1: 2
(5) 4 : 1 : 1
Solution:
P, Q and S contain 80%, 10% and 45% carbohydrates respectively.
To achieve 60% carbohydrates, proportion of P should be maximum. Hence, options
1 and 3 are eliminated.
Option 2:
Carbohydrate content
= (320 + 10 + 90)/700 = 420/700
= 60%
Cost per unit = (200 + 200 + 200)/700 = 6/7 = 0.857
Option 4:
Carbohydrate content
= (240 + 10 + 90)/600 < 60% Option 5: Carbohydrate content = (320 + 10 + 45)/600 = 62.5% Cost per unit = (200 + 200 + 100)/600 = 5/6 = 0. 833 P, Q and S in the proportion 4 : 1 : 1 has the lowest cost per unit. Hence, option 5. 50.The company is planning to launch a balanced diet required for growth needs of
adolescent children. This diet must contain at least 30% each of carbohydrate an d
protein, no more than 25% fat and at least 5% minerals. Which one of the following
combinations of equally mixed ingredients is feasible?
(1) O and P
(2) R and S
(3) P and S
(4) Q and R
(5) O and S
Solution:
A mixture of O and S in equal proportion satisfies the given constraints as can be
seen from the table.
Hence, option 5.
Carbohydrate Protein Fat Minerals
O & P 65 25 5 5
R & S 25 50 20 5
P & S 62.5 35 0 2.5
Q & R 7.5 40 45 7.5
O & S 47.5 40 5 7.5
Section III This section contains 25 questions

Directions for Questions 51 to 53: In each question, there are four sentences. Each
sentence has pairs of words/phrases that are italicized and highlighted. From the
italicized and highlighted word(s)/phrase(s), select the most appropriate
word(s)/phrase(s) to form correct sentences. Then, from the options given, choose
the best one.
51. The cricket council that was[A]/were[B] elected last March is[A]/are[B] at sixes
and sevens over new rules.
The critics censored[A]/censured[B] the new movie because of its social
inaccessibility.
Amit’s explanation for missing the meeting was credulous[A]/credible[B].
She coughed discreetly[A]/discretely[B] to announce her presence.
1) BBAAA
2) AAABA
3) BBBBA
4) AABBA
5) BBBAA
Solution:
The first sentence is the easiest to decide Since the ‘cricket council’ is singular
singular verbs (was and is) are required. Hence the answer choice should begin with
A. This eliminates option 1, 3, and 5.
Comparing option 2 and 4, the difference is in the third choice – credulous vs.
credible. Credulous means ready to believe easily and credible means: offering
reasonable grounds for being believed Hence Amit’s explanation is credible – B.
Sequence AAB is option 4.
Other confusable options: censor(v): to examine in order to delete anything that is
objectionable. Censure(v): criticize. Discrete: distinct; discreet: modest,
unnoticeable.
Hence, the correct answer is option 4.
52. The further[A]/farther[B] he pushed himself, the more disillusioned he grew.
For the crowds it was more of a historical[A]/historic[B] event; for their leader, it
was just another day.
The old man has a healthy distrust[A]/mistrust[B] for all new technology.
This film is based on a real[A]/true [B] story.
One suspects that the compliment[A]/complement[B] was backhanded.
1) BABAB
2) ABBBA
3) BAABA
4) BBAAB
5) ABABA
Solution:
Further is temporal and farther is spatial. The choice of A in the first sentence
eliminates option 1, 3 and 5.
Comparing 2 and 5 (ABBBA vs. ABABA), one can see that the third sentence is
decisive. Distrust: misgiving, lack or absence of trust. Mistrust: a lack of confidence
or uncertainty to regard as untrustworthy The tinge of ‘suspicion’ that colours
mistrust eliminates mistrust. Hence distrust is the correct use in the context.
Hence, the correct answer is option 5.
53. Regrettably[A]/Regretfully[B] I have to decline your invitation.
I am drawn to the poetic, sensual[A]/sensuous[B] quality of her paintings.
He was besides[A]/beside[B] himself with rage when I told him what I had done.
After brushing against a stationary[A]/stationery[B] truck my car turned turtle.
As the water began to rise over[A]/above[B] the danger mark, the signs of an
imminent flood were clear.
1) BAABA
2) BBBAB
3) AAABA
4) BBAAB
5) BABAB
Solution:
In this set, the last two sentences are the easiest to decide: stationary truck and to
rise above are correct uses. The answer choice has to end with AB. Options 1 and 3
are eliminated.
Beside oneself with rage is the correct idiom. Hence third sentence is B.
Sensuous implies gratification of the senses for the sake of aesthetic pleasure- the
sensuous delights of great music. Sensual tends to imply the gratification of the
senses or the indulgence of the physical appetites as ends in themselves – a life
devoted to sensual pleasures. Poetry is sensuous rather than sensual. Hence the
second sentence is B.
When we decline something we do it regretfully, when someone else has declined
we find it regrettable. Hence the first sentence is B.
Hence, the correct answer is option 2.
Directions for Questions 54 to 56: The passage given below is followed by a set of three
questions. Choose the most appropriate answer to each question.

To discover the relation between rules, paradigms, and normal science, consider first how
the historian isolates the particular loci of commitment that have been described as
accepted rules. Close historical investigation of a given specialty at a given time discloses a
set of recurrent and quasi-standard illustrations of various theories in their conceptual,
observational, and instrumental applications. These are the community’s paradigms,
revealed in its textbooks, lectures, and laboratory exercises. By studying them and by
practicing with them, the members of the corresponding community learn their trade. The
historian, of course, will discover in addition a penumbral area occupied by achievements
whose status is still in doubt, but the core of solved problems and techniques will usually
be clear. Despite occasional ambiguities, the paradigms of a mature scientific community
can be determined with relative ease.
That demands a second step and one of a somewhat different kind. When undertaking it,
the historian must compare the community’s paradigms with each other and with its
current research reports. In doing so, his object is to discover what isolable elements,
explicit or implicit, the members of that community may have abstracted from their more
global paradigms and deploy it as rules in their research. Anyone who has attempted to
describe or analyze the evolution of a particular scientific tradition will necessarily have
sought accepted principles and rules of this sort. Almost certainly, he will have met with at
least partial success. But, if his experience has been at all like my own, he will have found
the search for rules both more difficult and less satisfying than the search for paradigms.
Some of the generalizations he employs to describe the community’s shared beliefs will
present more problems. Others, however, will seem a shade too strong. Phrased in just
that way, or in any other way he can imagine, they would almost certainly have been
rejected by some members of the group he studies. Nevertheless, if the coherence of the
research tradition is to be understood in terms of rules, some specification of common
ground in the corresponding area is needed. As a result, the search for a body of rules
competent to constitute a given normal research tradition becomes a source of continual
and deep frustration.

Recognizing that frustration, however, makes it possible to diagnose its source. Scientists
can agree that a Newton, Lavoisier, Maxwell, or Einstein has produced an apparently
permanent solution to a group of outstanding problems and still disagree, sometimes
without being aware of it, about the particular abstract characteristics that make those
solutions permanent. They can, that is, agree in their identification of a paradigm without
agreeing on, or even attempting to produce, a full interpretation or rationalization of it.
Lack of a standard interpretation or of an agreed reduction to rules will not prevent a
paradigm from guiding research. Normal science can be determined in part by the direct
inspection of paradigms, a process that is often aided by but does not depend upon the
formulation of rules and assumption. Indeed, the existence of a paradigm need not even
imply that any full set of rules exists.
54. What is the author attempting to illustrate through this passage?
(1) Relationships between rules, paradigms, and normal science.
(2) How a historian would isolate a particular ‘loci of commitment’
(3) How a set of shared beliefs evolve in to a paradigm.
(4) Ways of understanding a scientific tradition.
(5) The frustrations of attempting to define a paradigm of a tradition
Solution:
Option 1 is factually correct and answers the question how rather than what.
Options 2, 3 and 5 are partial in answering the question what the author is trying to
illustrate.
Option 4 is supported by the following “To discover the relation between rules,
paradigms, and normal science, consider first how the historian isolates the
particular loci of commitment that have been described as accepted rules ” (at the
beginning of the passage) and “Normal science can be determined in part by the
direct inspection of paradigms formulation of rules and assumption ” (Towards
the end of the passage). This, then is the purpose of the passage.
Hence, the correct answer is option 4.
55. The term ‘loci of commitment’ as used in the passage would most likely correspond
with which of the following?
(1) Loyalty between a group of scientists in a research laboratory.
(2) Loyalty between groups of scientists across research laboratories.
(3) Loyalty to a certain paradigm of scientific inquiry.
(4) Loyalty to global patterns of scientific inquiry.
(5) Loyalty to evolving trends of scientific inquiry.
Solution:
The meaning given in option 3 to ‘loci of commitment’ is explicitly stated in the
passage The passage says the historian tries to isolate the ‘particular loci of
commitment’ at a given time and then explains what he is trying to find out and
concludes by saying ‘these are the community’s paradigms’ Thus loci of
commitment are the same as the paradigms.
None of the other options are worth evaluating because they are further in the
passage and not related to the question.
Hence, the correct answer is option 3.
56. The author of this passage is likely to agree with which of the following?
(1) Paradigms almost entirely define a scientific tradition.
(2) A group of scientists investigating a phenomenon would benefit by defining a set
of rules.
(3) Acceptance by the giants of a tradition is a sine qua non for a paradigm to
emerge.
(4) Choice of isolation mechanism determines the types of paradigm that may
emerge from a tradition.
(5) Paradigms are a general representation of rules and beliefs of a scientific
tradition.
Solution:
Option 5 is a mere definition of the term ‘paradigm’ as used in the passage
Paradigm in the context means a set of broad guidelines accepted by a group of
researchers. They are not as rigid as rules. They are not very concrete and differ
from community to community.
Option 1 is easily eliminated because of “entirely define’ which is too drastic
Option 2 is eliminated because of ’would benefit’ – nothing in the passage even
implicitly supports this.
Option 3 is contrary to the passage in the part referring to Newton, Lavoisier,
Maxwell, and Einstein.
Option 4 – ‘the choice of isolation mechanism’ is not discussed in the passage nor is
it even indirectly referr ed to.
Hence, the correct answer is option 5.
Directions for Questions 57 to 59: Each of the following questions has a paragraph from
which the last sentence has been deleted. From the given options, choose the sentence that
completes the paragraph in the most appropriate way.
57. Characters are also part of deep structure. Characters tie events in a story together
and provide a thread of continuity and meaning. Stories can be about individuals,
groups, projects or whole organizations, so from an organizational studies
perspective, the focal actor(s) determine the level and unit of analysis used in a
study. Stories of mergers and acquisitions, for example, are common place. In these
stories whole organizations are personified as actors. But these macro-level stories
usually are not told from the perspective of the macro-level participants, because
whole organizations cannot narrate their experiences in the first person.
(1) More generally, data concerning the identities and relationships of the
characters in the story are required, if one is to understand role structure and
social networks in which that process is embedded.
(2) Personification of a whole organization abstracts away from the particular
actors and from traditional notions of level of analysis.
(3) The personification of a whole organization is important because stories differ
depending on who is enacting various events.
(4) Every story is told from a particular point of view, with a particular narrative
voice, which is not regarded as part of the deep structure.
(5) The personification of a whole organization is a textual device we use to make
macro-level theories more comprehensible.
Solution:
The paragraph briefly is about why stories are structured around focal characters.
And why in stories of organizations, organizations have to be personified and focal
characters as organizations cannot narrate their experiences. Option 5 concludes
this chain of thoughts by saying that this kind of personification is a textual device
resorted to bring coherence.
Option 1 is incorrect as it continues the first part of the paragraph and is unrelated
the second part.
Option 2 is incorrect as it talks about abstracting away from the particular whereas
the paragraph is talking about particularizing.
Options 3 and 4 talk about different points of view, which is irrelevant to the
paragraph.
Hence, the correct answer is option 5.
58. Nevertheless, photographs still retain some of the magical allure that the earliest
daguerreotypes inspired. As objects, our photographs have changed; they have
become physically flimsier as they have become more technologically sophisticated.
Daguerre produced pictures on copper plates; today many of our photographs never
become tangible thins, but instead remain filed away on computers and cameras,
part of the digital ether that envelops the modern world. At the same time, our
patience for the creation of images has also eroded. Children today are used to being
tracked from birth by digital cameras and video recorders and they expect to see the
results of their poses and performances instantly. The space between life as it is
being lived and life as it is being displayed shrinks to a mere second.

(1) Yet, despite these technical developments, photographs still remain powerful
because they are reminders of the people and things we care about.
(2) Images, after all, are surrogates carried into battle by a soldier or by a traveller
on holiday.
(3) Photographs, be they digital or traditional, exist to remind us of the absent, the
beloved, and the dead.
(4) In the new era of the digital image, the images also have a greater potential for
fostering falsehood and trickery, perpetuating fictions that seem so re al we
cannot tell the difference.
(5) Anyway, human nature being what it is, little time has passed after
photography’s inventions became means of living life through images
Solution:
‘Nevertheless ‘at the beginning of the paragraph and “yet’ at the beginn ing
of option 1 make the paragraph logically complete.
Option 1 is the reason why the paragraph is written – to communicate that
‘photographs are still powerful’.
The traveler in option 2, the beloved and the dead in option 3, falsehood and
trickery in option 4, and the invention and means of living in option 5 do not help
conclude the paragraph.
Hence, the correct answer is option 1.
59. Mma Ramotswe had a detective agency in Africa, at the foot of Kgale Hill. These
were its assets; a tiny white van, two desks, two chairs, a telephone, and an old
typewriter. Then there was a teapot, in which Mma Ramotswe - the only private
lady detective in Botswana – brewed red bush tea. And three mugs – one for herself,
one for her secretary and one for the client. What else does a detective agency really
need? Detective agencies rely on human intuition and intelligence, both of which
Mma Ramotswe had in abundance.
(1) But there was also the view, which again would appear on no inventory.
(2) No inventory would ever include those, of course.
(3) She had an intelligent secretary too.
(4) She was a good detective and a good woman.
(5) What she lacked in possessions was more than made up by a natural
shrewdness.
Solution:
The paragraph mentions the tangible parts of the inventory that Mma Ramotswe
had at the agency, and ‘human intuition and intelligence’. Option 2 concludes the
paragraph by stating that ‘no inventory would ever be able to include those
Options 3, 4 and 5 are eliminated in comparison to options 1 and 2 which continue
the idea of the inventory.
Option 1, though continuing the idea of inventory is far inferior to option 2. The
‘those’ in option 2 scores over option 1
Hence, the correct answer is option 2.
Directions for Questions 60 to 62: The passage given below is followed by a set of three
questions. Choose the most appropriate answer to each question
The difficulties historians face in establishing cause-and-effect relations in the history of
human societies are broadly similar to the difficulties facing astronomers, climatologists,
ecologists, evolutionary biologists, geologists, and palaeontologists. To varying degrees
each of these fields is plagued by the impossibility of performing replicated, controlled
experimental interventions, the complexity arising from enormous numbers of variables,
the resulting uniqueness of each system, the consequent impossibility of formulating
universal laws, and the difficulties of predicting emergent properties and future behaviour.
Prediction in history, as in other historical sciences, is most feasible on large spatial scales
and over long times, when the unique features of millions of small-scale brief events
become averaged out. Just as I could predict the sex ratio of the next 1,000 newborns but
not the sexes of my own two children, the historian can recognize factors that made
inevitable the broad outcome of the collision between American and Eurasian societies
after 13,000 years of separate developments, but not the outcome of the 1960 U.S.
presidential election. The details of which candidate said what during a single televised
debate in October 1960 could have given the electoral victory to Nixon instead of to
Kennedy, but no details of who said what could have blocked the European conquest of
Native Americans.
How can students of human history profit from the experience of scientists in other
historical sciences? A methodology that has proved useful involves the comparative
method and so-called natural experiments. While neither astronomers studying galaxy
formation nor human historians can manipulate their systems in controlled laboratory
experiments, they both can take advantage of natural experiments, by comparing systems
differing in the presence or absence (or in the strong or weak effect) of some putative
causative factor. For example, epidemiologists, forbidden to feed large amounts of salt to
people experimentally, have still been able to identify effects of high salt intake by
comparing groups of humans who already differ greatly in their salt intake; and cultural
anthropologists, unable to provide human groups experimentally with varying resource
abundances for many centuries, still study long-term effects of resource abundance on
human societies by comparing recent Polynesian populations living on islands differing
naturally in resource abundance.
The student of human history can draw on many more natural experiments than just
comparisons among the five inhabited continents. Comparisons can also utilize large
islands that have developed complex societies in a considerable degree of isolation (such as
Japan, Madagascar, Native American Hispaniola, New Guinea, Hawaii, and many others), as
well as societies on hundreds of smaller islands and regional societies within each of the
continents. Natural experiments in any field, whether in ecology or human history, are
inherently open to potential methodological criticisms. Those include confounding effects
of natural variation in additional variables besides the one of interest, as well as problems
in inferring chains of causation from observed correlations between variables. Such
methodological problems have been discussed in great detail for some of the historical
sciences. In particular, epidemiology, the science of drawing inferences about human
diseases by comparing groups of people (often by retrospective historical studies), has for
a long time successfully employed formalized procedures for dealing with problems similar
to those facing historians of human societies.
In short, I acknowledge that it is much more difficult to understand human history than to
understand problems in fields of science where history is unimportant and where fewer
individual variables operate. Nevertheless, successful methodologies for analyzing
historical problems have been worked out in several fields. As a result, the histories of
dinosaurs, nebulae, and glaciers are generally acknowledged to belong to fields of science
rather than to the humanities.
60. Why do islands with considerable degree of isolation provide valuable insights into
human history?
(1) Isolated islands may evolve differently and this difference is of interest to us.
(2) Isolated islands increase the number of observations available to historians.
(3) Isolated islands, differing in their endowments and size may evolve differently
and this difference can be attributed to their endowments and size.
(4) Isolated islands, differing in their endowments and size, provide a good
comparison to large islands such as Eurasia, Africa, Americas and Australia.
(5) Isolated islands, in so far as they are inhabited, arouse curiosity about how
human beings evolved there.
Solution:
The answer is supported by the paragraph beginning “The student of human history
can draw on many more natural experiments than just comparisons among the five
inhabited continents. Comparisons can also utilize large islands that have developed
complex societies in a considerable degree of isolation as well as societies on
hundreds of smaller islands and regional societies within each of the continents ”
Options 1 and 3 say the same thing Option 1 is eliminated because the ‘difference’
mentioned in option 1 is explained in 3. Hence option 3 scores over option 1.
Option 4 is eliminated because ‘the good comparison to large islands’ is
inconsequential to the student of history. The student is more interested in knowing
how endowments and size affect societies – as a natural experiment.
Option 5 is eliminated because the paragraph says nothing about arousing ‘curiosity
about how humans evolved’ as stated in the option
Hence, the correct answer is option 3.
61. According to the author, why is prediction difficult in history?
(1) Historical explanations are usually broad so that no prediction is possible.
(2) Historical out comes depend upon a large number of factors and hence
prediction is difficult for each case.
(3) Historical sciences, by their very nature, are not interested in a multitude of
minor factors, which might be important in a specific historical outcome.
(4) Historians are interested in evolution of human history and hence are only
interested in log term predictions.
(5) Historical sciences suffer from the inability to conduct controlled experiments
and therefore have explanations based on a few long-term factors.
Solution:
This is directly stated in the passage “Prediction in history, as in other historical
sciences, is most feasible on large spatial scale and over long times, when the unique
future of millions of small scale brief events become averaged out ” The answer
option is merely the same thing expressed in different (even easier) words.
Option 1 is eliminated because of “the complexity arising from enormous numbers
of variables, the resulting uniqueness of each system, the consequent impossibility
of form ulating universal laws, and the difficulties of predicting emergent properties
and future behaviour” – in other words the explanations are broad because
prediction is not possible and not the other way round.
In option 3 ‘not interested’ is first data inadequate (passage does not say not
interested) and by implication incorrect because history is interested “ in a
multitude of minor factors ” – in order that the average may be worked out over
long periods of time.
Option 4 is factually correct but does not answer the question – why prediction is
difficult.
Option 5 is also factually correct but does not explain why prediction is difficult –it
merely explains the constraints that history faces and how then it operates.
Hence, the correct answer is option 2.
62. According to the author, which of the following statements would be true?
(1) Students of history are missing significant opportunities by not conducting any
natural experiments.
(2) Complex societies inhabiting large islands provide great opportunities for
natural experiments.
(3) Students of history are missing significant opportunities by not studying an
adequate variety of natural experiments.
(4) A unique problem faced by historians is their inability to establish cause and
effect relationships.
(5) Cultural anthropologists have overcome the problem of confounding variables
through natural experiments.
Solution:
The answer is directly supported by “The student of human history can draw on
many more natural experiments than the five inhabited continents Comparisons
can also utilize large islands as well as societies on hundreds of smaller islands
and regional societies within each of the continents” The implication is expressed in
option 3.
Option 1 is false in “not conducting ” – this is not true in the context of the passage,
nor is the author implying it.
Option 2 is false – the passage nowhere says that large islands provide ‘great’
opportunities for natural experiments” – they are one of the opportunities among
many.
The problem faced by historians is not ‘unique’ as stated in option 4 The passage
explicitly states that it is faced by several other studies mentioned in the first
sentence itself.
There is no data in the passage (even by implication) about cultural anthropologists.
Hence option 5 too is eliminated.
Hence, the correct answer is option 3.
Directions for Questions 63 to 65: In each question, there are five sentences or parts of
sentences that form a paragraph. Identify the sentence(s) or part(s) of sentence(s) that
is/are correct in terms of grammar and usage. Then, choose the most appropriate option.
63.
A. When I returned to home, I began to read
B. everything I could get my hand on about Israel.
C. That same year Israel’s Jewish Agency sent
D. a Shaliach a sort of recruiter to Minneapolis.
E. I became one of his most active devotees.
(1) C & E
(2) C only
(3) E only
(4) B, C & E
(5) C, D & E
Solution:
Statement A is incorrect because of the phrase ‘returned to home’ The correct
usage is ‘returned home’
Statement B is incorrect because the idiom is ‘get one’s hands on’ and not ‘hand on’.
Statement C is correct.
Statement D is incorrect because there should be a hyphen or a comma after a
Shaliach. (a Shaliach – a sort of recr uiter to Minneapolis ) “a sort of” though rather
informal, is correct usage.
Statement E is correct.
Hence, the correct answer is option 1.

64.
A. So once an economy is actually in recession,
B. The authorities can, in principle, move the economy
C. Out of slump – assuming hypothetically
D. That they know how to – by a temporary stimuli.
E. In the longer term, however, such policies have no affect on the overall behaviour of
the economy.
(1) A, B & E
(2) B, C & E
(3) C & D
(4) E only
(5) B only
Solution:
Statement A is incorrect – The use of the word ‘so’ is redundant and inappropriate
(So and once, in the context are adverbs – one of them is enough).
Statement B is correct.
Statement C is incorrect because the use of ‘assuming hypothetically’ makes it
redundant One can either assume or hypothesize but ‘assuming hypothetically’
is meaningless.
Statement D is incorrect in the plural use of ‘stimuli’ instead of ‘ stimulus’ with the
article ‘a’
Statement E should have been ‘effect’ instead of ‘affect’
Hence, the correct answer is option 5.
65.
A. It is sometimes told that democratic
B. government originated in the city-states
C. of ancient Greece. Democratic ideals have been handed to us from that time.
D. In truth, however, this is an unhelpful assertion.
E. The Greeks gave us the word, hence did not provide us with a model.
(1) A, B & D
(2) B, C & D
(3) B & D
(4) B only
(5) D only
Solution:
Statement A is incorrect because the verb ‘told’ is incorrectly used The verb
‘said’ should be used instead.
There is no error in statement B.
Statement C contains the incorrect idiom ‘handed to us’ instead of ‘handed down to
us’
Statement D is correct.
Statement E is incorrect because the word ‘hence’ is used as a conjunction whereas
it is an adverb. The use of a proper conjunction (e.g. and / but) will improve the
sentence.
Hence the correct answer is option 3.
Directions for Questions 66 to 68: The passage given below is followed by a set of three
questions. Choose the most appropriate answer to each question.
Human Biology does nothing to structure human society. Age may enfeeble us all, but
cultures vary consider ably in the prestige and power they accord to the elderly. Giving
birth is a necessary condition for being a mother, but it is not sufficient. We expect mothers
to behave in maternal ways and to display appropriately maternal sentiments. We
prescribe a clutch of norms or rules that govern the role of a mother. That the social role is
independent of the biological base can be demonstrated by going back three sentences.
Giving birth is certainly not sufficient to be a mother but, as adoption and fostering show, it
is not even necessary!
The fine detail of what is expected of a mother or a father or a dutiful son differs from
culture to culture, but everywhere behaviour is coordinated by the reciprocal nature of
roles. Husbands and wives, parents and children, employers and employees, waite rs and
customers, teachers and pupils, warlords and followers; each makes sense only in its
relation to the other The term ‘role’ is an appropriate one because the metaphor of an
actor in a play neatly expresses the rule-governed nature or scripted natur e of much of
social life and the sense that society is a joint production. Social life occurs only because
people play their parts (and that is as true for war and conflicts as for peace and love) and
those parts make sense only in the context of the overall show. The drama metaphor also
reminds us of the artistic licence available to the players. We can play a part straight or, as
the following from J.P. Sartre conveys, we can ham it up.

Let us consider this waiter in the cafe. His movement is quick and forward, a little too
precise, a little too rapid. He comes towards the patrons with a step a little too quick. He
bends forward a little too eagerly; his voice, his eyes express an interest a little too
solicitous for the order of the customer. Finally the re he returns, trying to imitate in his
walk the inflexible stiffness of some kind of automaton while carrying his tray with the
recklessness of a tightrope-walker….All his behaviour seems to us a game….But what is he
playing? We need not watch long before we can explain it: he is playing at being a waiter in
a cafe.

The American sociologist Erving Goffman built an influential body of social analysis on
elaborations of the metaphor of social life as drama. Perhaps his most telling point was that
it is only through acting out a part that we express character. It is not enough to be evil or
virtuous; we have to be seen to be evil or virtuous.

There is distinction between the roles we play and some underlying self. Here we might
note that some roles are more absorbing than others. We would not be surprised by the
waitress who plays the part in such a way as to signal to us that she is much more than her
occupation We would be surprised and offended by the father who played his part ‘tongue
in cheek’ Some roles are broader and more far-reaching than others. Describing someone
as a clergyman or faith healer would say far more about that person than describing
someone as a bus driver.
66. What is the thematic highlight of this passage?
(1) In the absence of strong biological linkages, reciprocal roles provide the
mechanism for coordinating human behaviour.
(2) In the absence of reciprocal roles, biological linkages provide the mechanism for
coordinating human behaviour.
(3) Human behaviour is independent of biological linkages and reciprocal roles.
(4) Human behaviour depends on biological linkages and reciprocal roles.
(5) Reciprocal roles determine normative human behavior in society.
Solution:
The theme of the passage is that biological linkages (for example mother – child;
father – child) do not structure human society. We expect a biological mother to
display certain characteristics in her ‘role’ as a mother as an ideal The passage is
then an explanation of how human society is structured on the basis of such
definitions of roles considered as ideals. Only option 5 captures this briefly.
Option 1 talks about ‘absence of strong biological linkages’ which is not dealt with in
the passage.
Option 2 is contrary to the theme of the passage.
Option 3 which states “ behavior is independent of reciprocal roles” is contrary
to the passage.
Option 4 may be evaluated as the answer, but passage does not state that human
behavior is dependent on biological linkages, and the example of the step mother
disproves this option.
Only option 5 captures the theme of the passage.
Hence, the correct answer is option 5.
67. Which of the following would have been true if biological linkages structured
human society?
(1) The role of mother would have been defined through her reciprocal relationship
with her children.
(2) We would not have been offended by the father playing his role ‘tongue in
cheek’
(3) Women would have adopted and fostered children rather than giving birth to
them.
(4) Even if warlords were physically weaker than their followers, they would still
dominate them.
(5) Waiters would have stronger motivation to serve their customers.
Solution:
“There is distinction between the roles we play and some underlying self Here we
might note that some roles are more absorbing than others. We would not be
surprised by the waitress who plays the part in such a way as to signal to us that she
is much more than her occupation. We would be surprised and offended by the
father who played his part ‘tongue in cheek’” (insincerely) The father’s self is
denied by his identification with his biological relationship. If this does not happen,
if a father behaves in a tongue in cheek manner, we are offended. If biological
relations structured human society, it is enough to be a biological father to be
accepted by society. His behavior (and with it, the reciprocal relationship) then
becomes unimportant.
All the other options support the fact that ‘reciprocal relationship’ structure human
society.
Hence, the correct answer is option 2.
68.It has been claimed in the passage that “some roles are more absorbing than others”
According to the passage, which of the following seem(s) appropriate reason(s) for
such a claim?
A. Some roles carry great expectations from the society preventing manifestation of
the true self.
B. Society ascribes so much importance to some roles that the conception of self
may get aligned with the roles being performed.
C. Some roles require development of skill and expertise leaving little time for
manifestation of self.
(1) A only
(2) B only
(3) C only
(4) A & B
(5) B & C
Solution:
The answer comes from the last paragraph where three examples are given, the
father, the waitress, and the priest.
The example of the priest makes statement A correct. (There is so much expectation
from the society that the priest’s true self is not revealed at all)
The father’s example makes statement B correct (The father’s self gets aligned with
his biological relationship and the self is denied).
Statement C is incorrect in that the passage does not discuss the development of
skill as a reason for the denial of the self.
Hence, the correct answer is option 4.
Directions for Questions 69 to 72: In each question, there are five
sentences/paragraphs. The sentence/ paragraph labelled A is in its correct place .
The four that follow are labelled B, C, D and E, and need to be arranged in the logical
order to form a coherent paragraph/passage. From the given options, choose the
most appropriate option.
69.
A. In America, highly educated women, who are in stronger position in the labour
market than less qualified ones, have higher rates of marriage than other groups.
B. Some work supports the Becker thesis, and some appears to contradict it.
C. And, as with crime, it is equally inconclusive.
D. But regardless of the conclusion of any particular piece of work, it is hard to
establish convincing connections between family changes and economic factors
using conventional approaches.
E. Indeed, just as with crime, an enormous academic literature exists on the validity of
the pure economic approach to the evolution of family structures.
(1) BCDE
(2) DBEC
(3) BDCE
(4) ECBD
(5) EBCD
Solution:
When the four statements are studied well, it is very easy to establish that EC and
BD are mandatory pairs.
Only statements E and C contain the idea of ‘crim e’ Hence one cannot place any
other statement along with statement E, but statement C.
In the same way, statements B and D both have reference to written ‘piece of work’ ,
making BD in that order mandatory. Once this is noticed, placing EC and BD in that
order with the help of A (fixed) is easy.
Hence, the correct answer is option 4.
70.
A. Personal experience of mothering and motherhood are largely framed in relation to
two discernible or “official” discourses: the “medical discourse and natural
childbirth discourse” Both of these tend to focus on the “optimistic stories” of birth
and mothering and underpin stereotypes of the “godmother”
B. At the same time, the need for medical expert guidance is also a feature for
contemporary reproduction and motherhood. But constructions of good mothering
have not always been so conceived – and in different contexts may exist in parallel to
other equally dominant discourses.
C. Similarly, historical work has shown how what are now taken-for-granted aspects of
reproduction and mothering practices result from contemporary “pseudoscientific
directives” and “managed constructs” These changes have led to a reframing of
modern discourses that pattern pregnancy and motherhood leading to an
acceptance of the need for greater expert management.
D. The contrasting, overlapping and ambiguous strands within these frameworks focus
to varying degrees on a woman’s biological tie to her child and predisposition to
instinctively know and be able to care for her child.
E. In addition a third “unofficial popular discourse” comprising “old wives” tales and
based on maternal experiences of childbirth has also been noted. These discourses
have also been acknowledged in work exploring the experiences of those who
apparently do not “conform” to conventional stereotypes of the “good mother”
(1) EDBC
(2) BCED
(3) DBCE
(4) EDCB
(5) BCDE
Solution:
The “two discernible” or “official discourses” makes it compulsory to place
statement E after statement A, because statement E talks about “a third unofficial
discourse”. (In other words if not placed next to statement A, statement E cannot be
placed anywhere else). AE is the first mandatory pair.
‘These frameworks’ in statement D is explained in statement E so that statement D
unless placed next to statement E, will not make sense. (In other words ED too is
mandatory.) The idea of motherhood from statement D (biological tie) is continued
in statement B. Thus the links in EDB are most obvious.
Statements C and B too are clearly linked because statement B ends with reference
to ‘dominant discourse’ and statement C begins with ‘historical work’ making EDBC
most logical sequence.
Hence, the correct answer is option 1.
71.
A. Indonesia has experienced dramatic shifts in its formal governance arrangements
since the fall of President Soeharto and the close of his centralized, authoritarian
“New Order” regime in 1997.
B. The political system has taken its place in the nearly 10 years since Reformasi began.
It has featured the active contest for political office among a proliferation of parties
at central, provincial and district levels; direct elections for the presidency (since
2004); and radical changes in centre -local government relations towards
administrative, fiscal, and political decentralization.
C. The mass media, once tidily under Soeharto’s thumb, has experienced significant
liberalization as has the legal basis for non-governmental organizations, including
many dedicated to such controversial issues as corruption control and human
rights.
D. Such developments are seen optimistically by a number of donors and some
external analysts, who interpret them as signs of Indonesia’s political normalization.
E. A different group of analysts paint a picture in which the institutional forms have
changed, but power relations have not. Vedi Hadiz argues that Indonesia’s
“democratic transition” has been anything but linear.
(1) BDEC
(2) CBDE
(3) CEBD
(4) DEBC
(5) BCDE
Solution:
As per the options comparing statements B, C and D as the sentences to follow
statement A, statement C gets eliminated. Statements B and D are far better
sentences to follow statement A than statement C.
The next decisive point is the ‘such developments’ in statement D As statements A
B, and C are talking about several developments, statement D is best placed at the
end of all, and will mar the structure of the paragraph if placed anywhere in
between.
The choice then becomes very clear. Also, the link between statement D and
statement E with their “some analysts’ (statement D) and ‘different analysts”
(statement E) is also obvious.
Hence, the correct answer is option 5.
72.
A. I had six thousand acres of land, and had thus got much spare land besides the
coffee plantation. Part of the farm was native forest, and about one thousand acres
were squatters’ land, what [the Kikuyu] called their shambas.
B. The squatters’ land was more intensely alive than the rest of the farm, and was
changing with the seasons the year round. The maize grew up higher than your head
as you walked on the narrow hard-trampled footpaths in between the tall green
rustling regiments.
unions
C. The squatters are Natives, who with their families hold a few acres on a white man’s
farm, and in return have to work for him a certain number of days in the year. My
squatters, I think, saw the relationship in a different light, for many of them were
born on the farm, and their fathers before them, and they very likely regarded me as
a sort of superior squatter on their estates.
D. The Kikuyu also grew the sweet potatoes that have a vine like leaf and spread over
the ground like a dense entangled mat, and many varieties of big yellow and green
speckled pumpkins.
E. The beans ripened in the fields, were gathered and thrashed by the women, and the
maize stalk and coffee pods were collected and burned, so that in certain seasons
thin blue columns of smoke rose here and there all over the farm.
(1) CBDE
(2) BCDE
(3) CBED
(4) DBCE
(5) EDBC
Solution:
Either by looking at the options or by reading the sentences in the given order, one
can easily see that statement A has to be followed either by statement B or by
statement C because they talk about the ‘squatters’ introduced in statement A (Th is
eliminates options 4 and 5).
A more careful reading of statement B and statement C establishes that since
statement C explains the identity of the squatters and statement B talks about their
farming, statement B has to follow statement C rather than precede it.
At his stage one has to evaluate/compare only options 1 and 3. Considering
statement E and statement D to follow statement B, the link between statement B
and statement E because of the “maize” conclusively makes option 3 the answer
Hence, the correct answer is option 3.

Directions for Questions 73 to 75: The passage given below is followed by a set of three
questions. Choose the most appropriate answer to each question.

Every civilized society lives and thrives on a silent but profound agreement as to what is to
be accepted as the valid mould of experience. Civilization is a complex system of dams,
dykes, and canals warding off, directing, and articulating the influx of the surrounding fluid
element; a fertile fenland, elaborately drained and protected from the high tides of chaotic,
unexercised, and inarticulate experience. In such a culture, stable and sure of itself within
the frontiers of ‘naturalized’ experience, the arts wield their creative power not so much in
width as in depth. They do not create new experience, but deepen and purify the old. Their
works do not differ from one another like a new horizon from a new horizon, but like a
madonna from a madonna.
The periods of art which are most vigorous in creative passion seem to occur when the
established pattern of experience loosens its rigidity without as yet losing its force. Such a
period was the Renaissance, and Shakespeare its poetic consummation. Then it was as
though the discipline of the old order gave depth to the excitement of the breaking away,
the depth of job and tragedy, of incomparable conquests and irredeemable losses.
Adventurers of experience set out as though in lifeboats to rescue and bring back to the
shore treasures of knowing and feeling which the old order had left floating on the high
seas. The works of the early Renaissance and the poetry of Shakespeare vibrate with the
compassion for live experience in danger of dying from exposure and neglect. In this
compassion was the creative genius of the age. Yet, it was a genius of courage, not of
desperate audacity. For, however elusively, it still knew of harbours and anchors, of homes
to which to return, and of barns in which to store the harvest. The exploring spirit of art
was in the depths of its consciousness still aware of a scheme of things into which to fit its
exploits and creations.

But the more this scheme of things loses its stability, the more boundless and uncharted
appears the ocean of potential exploration. In the blank confusion of infinite potentialities
flotsam of significance gets attached to jetsam of experience; for everything is sea,
everything is at sea -
…. The sea is all about us;
The sea is the land’s edge also, the granite
Into which it reaches, the beaches where it tosses
Its hints of earlier and other creation …
- and Rilke tells a story in which, as in T.S. Eliot’s poem, it is again the sea and the distance
of ‘other creation’ that becomes the image of the poet’s reality. A rowing boat sets out on a
difficult passage. The oarsmen labour in exact rhythm. There is no sign yet of the
destination. Suddenly a man, seemingly idle, breaks out into song. And if the labour of the
oarsmen meaninglessly defeats the real resistance of the real waves, it is the idle single
who magically conquers the despair of apparent aimlessness. While the people next to him
try to come to grips with the element that is next to them, his voice seems to bind the boat
to the farthest distance so that the farthest distance draws it towards itself. ‘I don’t know
why and how,’ is Rilke’s conclusion, ‘but suddenly I understood the situation of the poet,
his place and function in this age. It does not matter if one denies him every place - except
this one. There one must tolerate him.’
73.In the passage the expression “like a madonna from a madonna” alludes to
(1) The difference arising as a consequence of artistic license.
(2) The difference between two artistic interpretations.
(3) The difference between ‘life’ and ‘interpretation of life’
(4) The difference between ‘width’ and ‘depth’ of creative power
(5) The difference between the legendary character and the modern day singer.
Solution:
The lines, “ the art wield their creative power not so much in width as in depth
They do not create new experience, but deepen and purify the old. Their works do
not differ from one another like a new horizon from a new horizon ” tell us that the
works of art do not differ in their ‘width’ and ’depth’ (as mentioned in option 4) ‘life’
and its ‘interpretation’ (as mentioned in option 3) but are merely different
interpretations of the ‘old’ experience as one painting of Madonna (Virgin Mary the
mother of Jesus) differs from another version. This eliminates options 3 and 4.
The consequence of artistic license is not discussed in the passage so this eliminates
option 1.
Option 5 is ridiculous because the Madonna here does not refer to the modern day
singer.
Hence, the correct answer is option 2.
74. The sea and ‘other creation’ leads Rilke to
(1) Define the place of the poet in his culture.
(2) Reflect on the role of the oarsman and the singer.
(3) Muse on artistic labour and its aimlesseness.
(4) Understand the elements that one has to deal with.
(5) Delve into natural experience and real waves.
Solution:
Rilke’s conclusion is repeated almost verbatim in option 1 “I don’t know why and
how ’ is Rilke’s conclusion ‘but suddenly I understood the situation of the poet his
place and function in this age ” These are the concluding words of Rilke from the
passage after the example of the ‘sea’ and ‘the other creation’ mentioned in the
question.
Option 2 is mundane and quotes the example itself and not its purpose.
Option 3 is abstract and an ‘aimlessness’ cannot be attributed either to the oarsmen
or the singer.
Option 4 ‘understanding the elements’ is not the purpose of either the oarsmen or
the singer.
Option 5 is vague; the passage does not explain either natural experience or real
waves.
Hence, the correct answer is option 1.
75. According to the passage the term “adventurers of experience” refers to
(1) Poets and artists who are driven by courage.
(2) Poets and artists who create their own genre.
(3) Poets and artists of the Renaissance.
(4) Poets and artists who revitalize and enrich the past for us.
(5) Poets and artists who delve in flotsam and jetsam in sea.
Solution:
“Adventurers of experience set out as though in lifeboats to rescue and bring back to
the shore treasures of knowing and feeling which the old order had left floating on the
high seas. The work of the early Renaissance and the poetry of Shakespeare vibrate
with the compassion for live experience in danger of dying from exposure and
neglect. In this compassion was the creative genius of the age ” Renaissance artists
are cited as examples of ‘adventurers of experience’ These italicized words make
option 4 right.
As a result, option 3 is eliminated as being merely an example.
Driven by courage (option1), create their own genre (option 2) are partial and not
the intended meaning of the writer. This eliminates options 1 and 2.
Option 5 is also related to the example in a literal way, whereas the writer is being
symbolic in calling the artists adventurers. This eliminates option 5.
Hence, the correct answer is option 4.